Как найти полезная работа энергия

Вычисление полезной работы через КПД

Определение

Коэффициентом полезного действия (при сокращённом написании КПД) именуют безразмерную физическую величину, характеризующую отношение энергии, которую система потратила с пользой для нас, к полному количеству полученной энергии.

Измерять КПД принято в процентах. Например, КПД 35%, означает, что почти две трети энергии пошли на ненужные траты, стали рассеянным в пустую теплом, были потрачены на истирание деталей машины, образование искр и т. п.

Важно. 35% совсем не плохой КПД. У паровозов первой половины 20 века он составлял всего 10%. Лишь одна десятая образующегося при сгорании топлива тепла шла на перемещение состава, остальное рассеивалось в атмосфере. Среднеэксплуатационный КПД у современных тепловозов 20-22%. КПД машин на бензиновом ДВС равен 25%. КПД дизеля – 33%. Хорошо на этом фоне выглядит КПД электромобилей. Он у них около 90%.

В формуле нахождения полезной работы да в физике в основном КПД обозначают буквой из греческого алфавита η (эта).

Полезная работа в физике и ненужные траты энергии

Прежде чем говорить о том, как найти полезную работу в физике, следует сказать о ней самой. Дело в том что полезная работа в физике – величина очень даже субъективная. Она напрямую связана с человеческим восприятием, с тем, чего нам нужно получить от системы. Поэтому часто, когда говорят о КПД, имеют в виду различные технические устройства, а не природные объекты.

Хотя технологии постоянно развиваются избежать значительных потерь энергии всё же не удаётся. Получается, что:

Aзатр > Aполез

Aзатр – затраченная работа, Aполез – полезная работа, та что идёт на осуществление нужного нам процесса.

Как бы мы ни пытались уменьшить ненужные потери энергии, полностью от них избавиться не получиться. Непреодолимой преградой для этого является первый закон термодинамики. Из него явственно следует, что КПД любого устройства и механизма ни при каких обстоятельствах не может быть больше единицы и даже стать равным ей.

Формула

Общая формула КПД:

[η = (Aполез/Aзатр) * 100%].

Мощность представляет собой работу, совершённую за единицу времени. В связи с этим КПД можно посчитать как отношение входной мощности системы к выходной. Т. е.

η = Pвх/Pвых.

Как найти полезную работу в физике используя формулы для разных физических процессов

Вид формул, как найти полезную работу в физике, зависит от природы физических явлений, использующихся для преобразования затраченной энергии в нужную.

Нет времени решать самому?

Наши эксперты помогут!

Как найти полезную работу в физике механической системы

Лучше всего это показать на конкретном примере. Допустим, нам требуется найти КПД процесса, при котором мальчик вкатывает санки весом 4 кг на горку длиной 12, высотой 2 м. Он прикладывает для этого силу, равную 15 Н.

Решение:

Напомним, что общая формула для КПД

η = (Aполез/Aзатр) * 100%

Aполез в нашем случае равна потенциальной энергии (Eп), которую нужно потратить на то, чтобы поднять санки на высоту, т. е.  Aполез = m*g*h.

Затраченная мальчиком работа равна произведению силы на перемещение, т. е. Aзатр = F*S.

Подставляем в общую формулу для КПД

η = (m*g*h*100)/(F*S)

При подстановке численных значений получаем

η = 4*9,8*2/15*12 * 100% = 78,4/180 * 100% ≃ 43,6 %

Из этого примера ясно, чему равна полезная работа в физике механической системы, выраженная через КПД.

Формула

[Aполез = (η*F*S)/100]

Формула полезной работы в физике термодинамической системы

Именно по ней судят об эффективности тепловых машин. Допустим, нам нужно отыскать КПД тепловой машины, рабочее тело которой берёт от нагревателя 20кДж, а холодильнику отдаёт 10кДж.

Решение:

Тепловая машина работает следующим образом: нагреватель передаёт определённое количество теплоты рабочему телу, оно из-за этого расширяется, совершая тем самым механическую работу. Однако в последнюю переходит далеко не вся часть переданной тепловой энергии. Чтобы вернуть систему в исходное состояние и начать новый цикл приходится использовать холодильник.

Из выше сказанного можно сделать вывод, что Aполез равна разности энергии взятой от нагревателя и энергии, забранной холодильником, т. е.

Aполез = Qнагревателя – Qхолодильника

Затраченная работа равняется количеству той теплоты, которая была сообщена нагревателю.

Если всё это подставим в формулу для КПД, то получим

[eta=(text { Qнагревателя }-text { Qхолодильника })^{*} 100 / text { Qнагревателя }]

После подстановки численных значений будем иметь

η = (20 – 10)/20*100% = 50%

Теперь ясно, как определить полезную работу в физике термодинамической системы.

Формула

[eta=(text { Qнагревателя }-text { Qхолодильника })^{*} 100 / text { Qнагревателя }]

Формула полезной работы в физике электродинамической системы

Очень важный класс явлений. Каждый день все пользуются самыми разными электрическими устройствами: телевизором, компьютером, телефоном и т. д. Но мы рассмотрим случай попроще. Вычислим КПД электрического чайника. Допустим воде было передано 22176 Дж тепла за 2 мин. Напряжение в электросети стандартное 220 В. Сила тока равняется 1,4 А.

Решение:

Aполез будем считать работу, которая пошла на нагрев воды. Хотя она нам и дана из условия, формулу вспомнить всё равно не будет лишним.

  • Q = cm(tконечная-tначальная)
  • Q — количество теплоты [Дж]
  • c — удельная теплоёмкость вещества [Дж/кг*˚C]
  • m — масса [кг]
  • tконечная — конечная температура [˚C]
  • tначальная — начальная температура [˚C]
  • Работа тока вычисляется по формуле
  • A = (I^2)*Rt = (U^2)/R *t = UIt
  • A — работа электрического тока [Дж]
  • I — сила тока [А]
  • U — напряжение [В]
  • R — сопротивление [Ом]
  • t — время [c]

В нашем примере она примет вид

η = Q/A *100% = Q/UIt *100%

Переводим минуты в секунды и, подставляя численные значения, получаем

η = 22176/220*1,4*120 *100% = 60%

Формула полезной работы электродинамической системы будет:

Формула

[Aполез = (η*U*I*t)/100%]

Автор статьи

Сергей Феликсович Савельев

Эксперт по предмету «Физика»

Задать вопрос автору статьи

Определение 1

КПД (коэффициент полезного действия) — величина, характеризующая
соотношение используемой энергии к затрачиваемой, т.е. энергетическую эффективность системы.

КПД измеряется в процентах или указывается как десятичная дробь от 0 до 1. КПД 50% (или, что тоже самое– 0,5) означает, что только половина энергии используется для выполнения работы. Остальная рассеивается в окружающем пространстве, как правило, в форме тепла.

Замечание 1

Коэффициент полезного действия паровозов, применявшихся для железнодорожных перевозок в XIX — первой половине XX вв., составлял менее 10%, т.е. 90 и более процентов тепла от сжигаемого в топках угля улетучивалось в атмосферу, не выполняя полезной работы по вращению колес, приводящему к движению состав. Для сравнения: КПД пришедших на смену паровозам тепловозов (в них используются не паровые, а дизельные двигатели) достигает 40%.

КПД в формулах обозначают греческой буквой $eta$ (эта).

$eta = frac{A_п}{A_з}$

, где $A_п$ — полезная работа, $A_з$ — затраченная.

Полезная работа и потери энергии

«Полезность» выполняемой работы — величина субъективная, связанная с человеческим восприятием, поэтому о КПД говорят чаще всего применительно к искусственно созданным системам. Несмотря на то, что технологии совершенствуются, избежать потерь в рукотворных системах инженерам не удастся:

  • в механических устройствах часть затрачиваемой энергии всегда тратится на преодоление сил трения между соприкасающимися деталями (эти силы уменьшают за счет более тщательной обработки и смазки);
  • в электрических системах часть энергии рассеивается в виде тепла при преодолении сопротивления проводников (явление сверхпроводимости еще не применимо к практике и требует низких температур);
  • в нагревательных приборах утечки происходят в силу дефектов теплоизоляции и т.п.

Таким образом,

$A_з$ > $A_п$

, где $A_з$ — работа затраченная, $A_п$ — работа полезная.

«Формула полезной работы в физике для КПД» 👇

Потери энергии можно сводить к минимуму, но полностью исключить их невозможно. Какое бы совершенное устройство мы не придумали, КПД никогда не достигнет единицы в силу второго закона термодинамики, действие которого исключает создание механизмов с КПД равным или большим 100%.

КПД различных физических процессов

Методики подсчета КПД разнятся в зависимости от физической природы явлений, задействованных в преобразующих энергию системах.

При практических расчетах, связанных с движением, знаменатель формулы КПД удобнее представить не как работу (произведение силы на расстояние), а как затраченную энергию, выделившуюся, например, при сжигании топлива:

$eta = frac{A_п}{Q}$

, где $A_п$ — выполненная системой полезная работа, $Q$ — затраченная системой энергия.

Например, зная сколько бензина истрачено двигателем автомобиля (количество выделившегося в результате тепла можно легко подсчитать), а также массу, скорость и пройденное расстояние, легко найти КПД.

Если речь идет не об автомобиле с двигателем внутреннего сгорания, а об электромобиле, то затраты энергии в знаменателе можно подсчитать как произведение средних тока и напряжения за время движения рассматриваемого транспортного средства.

Поскольку мощность представляет собой работу, выполняемую в единицу времени, КПД иногда бывает удобно посчитать как соотношение входной и выходной мощностей системы:

$eta = frac{P_{out}}{P_{in}}$

, где $P_{in}$ — мощность на входе системы, $P_{out}$ — на выходе.

Такой подход удобен, например, при расчете КПД солнечных батарей. В знаменателе в этом случае будет мощность светового излучения, падающего на их поверхность, в числителе — мощность генерируемого тока.

Пример 1

Лебедка, потребляющая мощностью 500 Вт, за время 10 с подняла груз массой 70 кг на высоту 5м. Найти КПД лебедки.

Лебедка преодолела силу тяжести, совершив работу

$A_л = m cdot g cdot h$

, где $m$ — масса, $g$ — ускорение свободного падения, $h$ высота.

Подставив значения, получаем:

$A = 70 cdot 9,8 cdot 5 = 3430 Дж$

Затраченную энергию найдем через мощность и время:

$Q = P cdot t$

, где $Q$ — энергия, $P$ — мощность, $t$ — время.

Подставив значения, получаем:

$Q = 500 Вт cdot 10 с = 5000 Дж$

КПД находим как соотношение

$eta = frac{A}{Q} = frac{3430}{5000}cdot 100$% = $68,6$%

Ответ: КПД лебедки равен 68,6%.

Находи статьи и создавай свой список литературы по ГОСТу

Поиск по теме

Полезная работа и потери энергии

“Полезность” выполняемой работы – величина субъективная, связанная с человеческим восприятием, поэтому о КПД говорят чаще всего применительно к искусственно созданным системам. Несмотря на то, что технологии совершенствуются, избежать потерь в рукотворных системах инженерам не удастся:

  • в механических устройствах часть затрачиваемой энергии всегда тратится на преодоление сил трения между соприкасающимися деталями (эти силы уменьшают за счет более тщательной обработки и смазки);
  • в электрических системах часть энергии рассеивается в виде тепла при преодолении сопротивления проводников (явление сверхпроводимости еще не применимо к практике и требует низких температур);
  • в нагревательных приборах утечки происходят в силу дефектов теплоизоляции и т.п.

Таким образом,

$A_з$ > $A_п$

, где $A_з$ – работа затраченная, $A_п$ – работа полезная.

Потери энергии можно сводить к минимуму, но полностью исключить их невозможно. Какое бы совершенное устройство мы не придумали, КПД никогда не достигнет единицы в силу второго закона термодинамики, действие которого исключает создание механизмов с КПД равным или большим 100%.

КПД различных физических процессов

Методики подсчета КПД разнятся в зависимости от физической природы явлений, задействованных в преобразующих энергию системах.

При практических расчетах, связанных с движением, знаменатель формулы КПД удобнее представить не как работу (произведение силы на расстояние), а как затраченную энергию, выделившуюся, например, при сжигании топлива:

$eta = frac{A_п}{Q}$

, где $A_п$ — выполненная системой полезная работа, $Q$ — затраченная системой энергия.

Например, зная сколько бензина истрачено двигателем автомобиля (количество выделившегося в результате тепла можно легко подсчитать), а также массу, скорость и пройденное расстояние, легко найти КПД.

Если речь идет не об автомобиле с двигателем внутреннего сгорания, а об электромобиле, то затраты энергии в знаменателе можно подсчитать как произведение средних тока и напряжения за время движения рассматриваемого транспортного средства.

Поскольку мощность представляет собой работу, выполняемую в единицу времени, КПД иногда бывает удобно посчитать как соотношение входной и выходной мощностей системы:

$eta = frac{P_{out}}{P_{in}}$

, где $P_{in}$ – мощность на входе системы, $P_{out}$ – на выходе.

Такой подход удобен, например, при расчете КПД солнечных батарей. В знаменателе в этом случае будет мощность светового излучения, падающего на их поверхность, в числителе – мощность генерируемого тока.

Пример 1

Лебедка, потребляющая мощностью 500 Вт, за время 10 с подняла груз массой 70 кг на высоту 5м. Найти КПД лебедки.

Лебедка преодолела силу тяжести, совершив работу

$A_л = m cdot g cdot h$

, где $m$ – масса, $g$ – ускорение свободного падения, $h$ высота.

Подставив значения, получаем:

$A = 70 cdot 9,8 cdot 5 = 3430 Дж$

Затраченную энергию найдем через мощность и время:

$Q = P cdot t$

, где $Q$ – энергия, $P$ – мощность, $t$ – время.

Подставив значения, получаем:

$Q = 500 Вт cdot 10 с = 5000 Дж$

КПД находим как соотношение

$eta = frac{A}{Q} = frac{3430}{5000}cdot 100$% = $68,6$%

Ответ: КПД лебедки равен 68,6%.

Механическая работа – это одна из основных скалярных величин в физике. В рамках стандартной школьной программы она изучается в седьмом классе в разделе механики. Механическая работа – один из способов изменения внутренней энергии тела или субстанции (например, газа или жидкости) наряду с такими формами теплопередачи, как теплопроводность, конвекция и излучение, которые изучаются в разделе тепловых явлений.

Что такое работа в физике – определение и формула

Механическая работа – это количество энергии, которое нужно затратить для того, чтобы тело начало равномерно замедляющееся движение и прошло некоторую дистанцию. 

В физике механической работой называется произведение силы, которая действует на некоторое тело, на расстояние, которое оно проходит под ее воздействием:

A = F * S

В более сложных случаях в формуле появляется и третья величина – косинус угла, под которым друг к другу расположены векторы движения и приложенной силы. Найти ее значение можно по формуле:

A = F * S * cosA

В чем измеряется работа

Физические единицы, в которых выражается механическая работа, – Джоули. 

Существуют разные способы для ее практического измерения, которые зависят от типа произведенного движения. При этом в формулу работы подставляют значение силы в Ньютонах и расстояния в метрах. Угол между векторами измеряют в математических единицах – градусах. 

Работа силы трения

При условиях, существующих на Земле, на любое движущееся тело оказывает воздействие сила трения, замедляющая его движение. Чаще всего это трение поверхности, по которой движется объект. Это очевидно из того факта, что при воздействии постоянной силы на тело его скорость окажется переменной. 

Следовательно, должна быть и другая сила, противодействующая ей – и это сила трения. Если система координат выбрана по направлению движения тела, то ее числовое значение будет отрицательным.

Положительная и отрицательная работа

Числовое значение работы, которую совершает сила, может становиться отрицательным в случае если ее вектор противоположен вектору скорости. 

Иными словами, сила может не только придавать телу скорость для совершения движения, но и препятствовать уже совершаемому перемещению. В таком случае она будет называться противодействующей. 

Полезная или затраченная работа

У тела, совершающего одно и то же действие, есть два значения работы. Первая из них, полезная, вычисляется по обычной формуле. 

Вторая, затраченная, по своему понятию не имеет общей формулы для вычисления и измеряется практически. Эта разница между совершенной в реальности работой и той, которая должна была быть совершена в теории, равна коэффициенту полезного действия – КПД. Он вычисляется так:

КПД = А полезная / А затраченная,

и выражается в процентах. КПД всегда меньше 100.

Мощность

Среднее количество работы, совершаемой за единицу времени (секунду), характеризует такую величину, как мощность. Формула для ее вычисления выглядит так:

Р = A / t

В качестве работы можно подставить люблю известную формулу для ее вычисления в зависимости от ситуации. Ответ будет выражен в Ваттах.

Однако при равномерном движении можно использовать и другую формулу:

Р = F * v

Подставив вместо обычной скорости мгновенную, можно получить значение мгновенной мощности.

Примеры решения задач

Рассмотрим несколько простых задач на нахождение механической работы.

Задача 1

Какую работу совершает подъемный механизм, поднимающий десятикилограммовый блок на высоту 50 метров.

Решение:

Для того, чтобы поднять тело, необходимо преодолеть действующую на него силу тяжести. То есть F, с которой поднимают блок, равна той, с которой он притягивается к земле. Так как последняя равна m * g, то для нахождения конечного результата понадобится только одна измененная версия стандартной формулы, упомянутой выше: A = S * m * g.

При помощи простой математики найдем числовой ответ:

A = 50 м * 10 кг * 10 Н/кг;

A = 5000 Дж.

Ответ: 5000 Дж.

Впрочем, не всегда речь идет о силе тяжести.

Задача 2

Какая работа совершается силой упругости, когда пружина с жесткостью 10 Н/м, сжатая на 20 см, возвращается в исходное состояние? Система замкнута, нет никаких внешних сил, воздействующих на пружину.

Решение:

Для начала нужно найти саму F упругости, которая совершает работу. Ее формула – F = x * |k|, где x – это длина, на которую сжимается или растягивается пружина, а k – коэффициент ее жесткости. Перемещение пружины равно ее деформации, и следовательно, конечная формула в этом случае будет выглядеть так: A = S * x * k = x * x * k = x^2 * k.

Далее при помощи элементарных вычислений рассчитаем ответ:

A = (0,2 м)^2 * 10 Н/м = 0,04 * 10 = 0,4 Дж.

Ответ: 0,4 Дж.

Но во всех задачах по данной теме траектория движения тела прямая.

Задача 3

Рассчитайте, какова сила, действующая на колесо, если на то, чтобы совершить полный оборот, ему требуется 10 кДж. Диаметр диска равен 40 см, а толщина шины – 10 см.

Решение:

В этом случае нам нужно найти не А, а F, но сделать это можно при помощи все той же формулы. Возьмем точку на поверхности колеса. Предположим, что при вращательном движении ее вектор будет противоположен вектору приложения силы, а значит косинусом в формуле вновь можно пренебречь. Таким образом, за один оборот колеса точка пройдет расстояние, равное длине окружности, которую можно вычислить как 2πr или πd. Диаметр окружности можно найти из предоставленных данных: он равен сумме диаметра диска и удвоенной толщины шины, то есть 40 см + 2 * 10 см = 40 см + 20 см = 60 см = 0,6 м.

Теперь, когда мы можем вычислить расстояние, у нас есть все данные для того, чтобы приступить к нахождению силы.

Формула работы для этого случая будет такой: A = F * π * d, то силу, соответственно, можно будет выразить как F = A / (π * d).

В таком случае:

F = 10 кДж / (3,14 * 0,6 м) = 10000 Дж / 1,884 м = ~ 5308 Н.

Ответ: 5308 Н.

В завершение решим самый сложный вариант задачи, включающий в себя все, о чем говорилось выше.

Задача 4

Автомобиль Фольксваген весом 2500 кг заезжает на гору. Какова должна быть его минимальная скорость, чтобы удержаться на горе, если сила тяги равна 10 кН, время работы двигателя – 10 с, КПД – 30%, а угол наклона горы – 60 градусов. Трением и прочими силами пренебречь.

Решение:

На первый взгляд задача может показаться сложной, но для ее решения используются только простые известные формулы. 

Запишем условие в более наглядном виде.

Дано:

m = 2500 кг;

F = 10000 H;

t = 10 с;

КПД = 30%;

угол A = 1500 (60+90, т. к. сила тяжести приложена под углом 90 к горизонтали);

V – ?

Выведение формулы:

Шаг 1. По условию A1 (силы тяжести) = А2 (тяги).

A1 = mg;

A2 = P * t / КПД.

То есть mg = P * t / КПД.

Шаг 2. P = F * V * cosA.

Шаг 3. Общая формула: mg = F * V * cosA * t / КПД.

V = (m * g * КПД) / (F * t * cosA).

Числовое решение:

V = (2500 кг * 10 Н/кг * 30%) / (10000 H * 10 с * cos150);

V = (2500 кг * 10 Н/кг * 0,3) / (10000 H * 10 с * cos60);

V = 7500 / 50000;

V = 0,15 м/с.

Ответ: 0,15 м/с.

Источник

Определение и расшифровка КПД

Расшифровка аббревиатуры — коэффициент полезного действия. Однако и такое толкование с первого раза может оказаться не особо понятным. Этим коэффициентом характеризуется эффективность системы или какого-либо отдельного тела, а чаще — механизма. Эффективность характеризуется отдачей или преобразованием энергии.

Этот коэффициент применим практически ко всему, что нас окружает, и даже к нам самим, причём в большей степени. Ведь совершаем мы полезную работу все время, только вот как часто и насколько это важно, уже другой вопрос, с ним и используется термин «КПД».

Важно учесть, что этот коэффициент — величина неограниченная, она, как правило, представляет собой либо математические значения, к примеру, 0 и 1, либо же, как это чаще бывает — в процентах.

В физике этот коэффициент обозначается буквой Ƞ, или, как её привыкли называть, Эта.

Полезная работа

КПД — это отношение полезной работы, совершенной машиной, ко всей затраченной работе (подведенной энергии) за то же время

Коэффициент полезного действия выражается в процентах. Чем ближе это число к 100%, тем выше производительность машины. Не может быть КПД больше 100, так как невозможно выполнить больше работы, затратив меньше энергии.

Читайте также:  Контрольная на тему налог на добычу полезных ископаемых

КПД наклонной плоскости — это отношение работы силы тяжести, к затраченной работе по перемещению вдоль наклонной плоскости.

При использовании каких-либо механизмов или устройств мы обязательно совершаем работу. Она, как правило, всегда больше той, что необходима нам для выполнения поставленной задачи. Исходя из этих фактов различается два типа работы: это затраченная, которая обозначается большой буквой, А с маленькой з (Аз), и полезная — А с буквой п (Ап).

В случае когда для подъема применяется какое-либо устройство, кроме силы тяжести булыжника, важно учесть еще и силу тяжести частей этого устройства. И кроме всего этого, важно помнить, что, выигрывая в силе, мы всегда будем проигрывать в пути. Все эти факты приводят к одному выводу, что затрачиваемая работа в любом варианте окажется больше полезной, Аз gt;

Полезная работа — это часть затрачиваемой, которую мы совершаем, используя механизм. А КПД — это как раз та физическая величина, которая показывает, какую часть составляет полезная работа от всей затраченной.

  • Затрачиваемая работа Aз всегда больше полезной Ап.
  • Чем больше отношение полезной к затрачиваемой, тем выше коэффициент, и наоборот.
  • Ап находится произведением массы на ускорение свободного падения и на высоту подъема.

Физическая формула КПД

Существует определенная формула для нахождения КПД. Она звучит следующим образом: чтобы найти КПД в физике, нужно количество энергии разделить на проделанную системой работу. То есть КПД — это отношение затраченной энергии к выполненной работе. Отсюда можно сделать простой вывод, что тем лучше и эффективнее система или тело, чем меньше энергии затрачивается на выполнение работы.

Сама формула выглядит кратко и очень просто Ƞ будет равняться A/Q. То есть Ƞ = A/Q. В этой краткой формулы и фиксируют нужные нам элементы для вычисления. То есть A в этом случае является использованной энергией, которая потребляется системой во время работы, а большая буква Q, в свою очередь, будет являться затраченной A, или опять же затраченной энергией.

В идеале КПД равен единице. Но, как это обычно бывает, он её меньше. Так происходит по причине физики и по причине, конечно же, закона о сохранении энергии.

Все дело в том, что закон сохранения энергии предполагает, что не может быть получено больше А, чем получено энергии. И даже единице этот коэффициент будет равняться крайне редко, поскольку энергия тратится всегда. И работа сопровождается потерями: к примеру, у двигателя потеря заключается в его обильном нагреве.

  • A — полезная работа, которую выполняет система.
  • Q — энергия, которую потребляет система.

Применение в разных сферах физики

Примечательно, что КПД не существует как понятие нейтральное, для каждого процесса есть свой КПД, это не сила трения, он не может существовать сам по себе.

Рассмотрим несколько из примеров процессов с наличием КПД.

К примеру, возьмем электрический двигатель. Задача электрического двигателя — преобразовывать электрическую энергию в механическую. В этом случае коэффициентом будет являться эффективность двигателя в отношении преобразования электроэнергии в энергию механическую. Для этого случая также существует формула, и выглядит она следующим образом: Ƞ=P2/P1. Здесь P1 — это мощность в общем варианте, а P2 — полезная мощность, которую вырабатывает сам двигатель.

Нетрудно догадаться что структура формулы коэффициента всегда сохраняется, меняются в ней лишь данные, которые нужно подставить. Они зависят от конкретного случая, если это двигатель, как в случае выше, то необходимо оперировать затрачиваемой мощностью, если работа, то исходная формула будет другая.

Теперь мы знаем определение КПД и имеем представление об этом физическом понятии, а также об отдельных его элементах и нюансах. Физика — это одна из самых масштабных наук, но её можно разобрать на маленькие кусочки, чтобы понять. Сегодня мы исследовали один из этих кусочков.

Полезная работа и затраченная работа формулы

На этом уроке узнаем, куда пропадает часть затраченной энергии при работе каких-либо механизмов, научимся решать задачи с использованием КПД и познакомимся с мерами по увеличению КПД разных механизмов.

Полезная и затраченная работа

На предыдущих занятиях при рассмотрении устройства и работы простейших механизмов мы не учитывали трение между деталями механизмов, вес механизмов – это идеализированные условия. На практике работа, совершаемая приложенной к телу силой, называется затраченной, она всегда больше работы, которая совершается по перемещению груза, поднятию груза или преодолению сопротивления, эта работа называется полезной (Рис. 1). Полезная работа меньше затраченной

Понятие КПД (коэффициента полезного действия)

Термин «КПД» широко используется не только среди профессионалов, но и в быту. Под ним понимают, насколько совершенная работа превышает полезную, т.е. ту, ради которой механизм или прибор приобретается.

Учеными разработана специальная формула, из которой следует, что КПД всегда меньше единицы. Чтобы рассчитать коэффициент, нужно полезную работу, выраженную в Джоулях, разделить на энергию, которая затрачена на эту работу. Поскольку энергия также выражается в Джоулях, конечная расчетная величина безразмерна.


Объяснить бытовым языком данное понятие можно так: энергия, выделяемая от плиты, на которой должен закипеть чайник, расходуется не только на его нагревание. Она должна нагреть саму посудину, воздух вокруг нее, сам нагревательный элемент. И только ее часть будет расходоваться на передачу воде. Чтобы сориентироваться, насколько долго будет закипать чайник одного объема на различного вида печах, нужно знать их КПД.

В поисках наиболее эффективного прибора не стоит стремиться к единице. Такого не бывает. Например, КПД атомной электростанции примерно равно 35%.

Происходит это по двум причинам:

  1. Исходя из закона сохранения энергии, получить больше работы, чем затрачено энергии, невозможно.
  2. Любая работа сопровождается определенными потерями, будь-то нагревание тары или преодоление сил трения при движении по поверхности.

Термин КПД применим практически к каждому процессу, в котором имеется затраченная и полезная работа.

Применение в различных сферах физики

Характеризуя КПД, следует учитывать, что он не является константой, поскольку в каждом случае свои особенности энергозатрат. С другой стороны, он не может быть установлен изолированно от конкретных процессов. Если рассмотреть работу электродвигателя, величина его КПД сложится исходя из преобразования энергии тока в механическую работу.

В данном случае КПД рассматривается не как соотношение полезной и общей работы, а как соотношение отдаваемой мощности и подводимой к рабочему механизму.

В формулу (η=P2/P1) должны быть включены P1 – первичная мощность и P2 – мощность прибора.

В качестве первого примера выведем формулу КПД для варианта определения с величинами работы и затраченной энергии (формула для определения КПД теплового двигателя). Условными обозначениями в ней будут являться:
Ап – работа полезная;

  • Q1 – количество энергии (или тепла), полученной от нагревающего устройства;
  • Q2 – количество энергии (или тепла), отданное в процессе деятельности;
  • Q1 – Q2 – та энергия (или тепло), которая пошла на процесс.

В итоге получится выражение:

Теперь выразим формулу через соотношение мощностей. Условные обозначения следующие:

Ротд – полезная (эффективная) мощность ;

Рподв – номинальная мощность.

Формула будет выглядеть так:

Если затрата или передача энергии происходит неоднократно, общий КПД равен сумме КПД на каждом участке процесса:

Какой буквой обозначается, единицы измерения

В вышеприведенной формуле искомая величина коэффициента полезного действия обозначается буквой η, которая произносится «эта».

Для упрощения понимания величины, КПД чаще выражается в процентах.

Физическая формула КПД

С учетом изложенных выше особенностей и необходимости выражения результата в %, физические формулы приобретают усовершенствованный внешний вид:

Примеры расчета КПД

Формула применяется для расчетов коэффициентов машин различного типа.

Задача 1

Имеется 10 кг дров, теплота сгорания которых составляет 95 Дж/кг. При их сгорании в помещении объемом 75 м3 установилась температура 22оС (допускаем, что удельная теплоемкость воздуха равна 1,3 кДж/ кгхград).

Решение состоит из нескольких действий:

  1. 1300 Дж умножить на 75 (объем) и 22 (температуру). Получаем 2 145 кДж. Это то тепло, выраженное в кДж, которое поступило в воздух помещения.
  2. 10700000Дж умножаем на 10 (количество дров) =10х107 кДж.
  3. При делении полезного тепла и полного, выработанного обогревателем, получаем значение 2,5%. Это говорит о низкой эффективности прибора и большой затрате дров и необходимости внесения конструктивных изменений, например, оборудования возможности дымоходам нагревать не только воздух, но и предметы в помещении.

Задача 2

В доме установлен электробойлер объемом 80 литров. Нагревательный элемент имеет мощность 2 кВт. Было замечено, что для нагревания воды от 12 о С до 70 о С уходит 3 часа. Нужно определить КПД прибора.

Дополнительные данные: плотность воды составляет 1000 кг/м 3 , ее теплоемкость – 4200 Дж/кг* о С.

Решать задачу нужно по формуле:

(eta=(ctimesrhotimes Vtimes(T_2-T_1)div Ntimes t)times100%=90%)

Задача 3

Температура воды, налитой в котел паровой машины, составляет 160 о С. Температура холодильника – 10 о С. Коэффициент полезного действия машины – 60%. В топке сжигается 200 кг угля. Его удельная теплота сгорания – 2,9 • 107 Дж/кг. О какой максимальной работе может идти речь для данной машины?

Решение следующее. Амакс возможна для идеальной тепловой машины, которая функционирует по циклу Карно. Ее КПД равно (Т1-Т2)/Т1. В этой формуле Т1 и Т2 – температуры нагревателя, холодильника.

Читайте также:  Прибор для измерения световой температуры

Определяем КПД, пользуясь формулой: ( eta;=;Adiv Q_1) . В этой формуле А – работа тепловой машины, Q1 – теплота, полученная от нагревателя. С другой стороны, она равна (eta_1times mtimes q) .

(Q_1;=;eta_1times mtimes q)

((T_1-T_2)div T_1=Adiveta_1times mtimes g)

(А;=;eta_1times mtimes qtimes(1;-;Т_2div Т_1))

Подставив значение, получаем ответ: 1,2*109 Дж.

Работа, мощность, КПД

Сила, перемещающая тело, совершает работу. Работа – это разность энергии тела в начале процесса и в его конце. А мощность – это работа за одну секунду. Коэффициент полезного действия (КПД) – это дробное число. Максимальный КПД равен единице, однако, часто, КПД меньше единицы.

Работы силы, формула

Сила, приложенная к телу и перемещающая его, совершает работу (рис. 1).

Работа силы — это скалярное произведение вектора силы на вектор перемещения.

Работу, совершаемую силой, можно посчитать, используя векторный или скалярный вид записи такой формулы:

Векторный вид записи

Для решения задач правую часть этой формулы удобно записывать в скалярном виде:

[ large boxed < A = left| vecright| cdot left| vec right| cdot cos(alpha) >]

( F left( H right) ) – сила, перемещающая тело;

( S left( text <м>right) ) – перемещение тела под действием силы;

( alpha ) – угол между вектором силы и вектором перемещения тела;

Работу обозначают символом (A) и измеряют в Джоулях. Работа – это скалярная величина.

В случае, когда сила постоянная, формула позволяет рассчитать работу, совершенную силой за полное время ее действия.

Если сила изменяется со временем, то в каждый конкретный момент времени будем получать мгновенную работу. Эти, мгновенные значения для разных моментов времени будут различаться.

Рассмотрим несколько случаев, следующих из формулы:

  1. Когда угол между силой и перемещением острый, работа силы положительная;
  2. А если угол тупой — работа отрицательная, так как косинус тупого угла отрицательный;
  3. Если же угол прямой – работа равна нулю. Сила, перпендикулярная перемещению, работу не совершает!

Работа — разность кинетической энергии

Работу можно рассчитать еще одним способом — измеряя кинетическую энергию тела в начале и в конце процесса движения. Рассмотрим такой пример. Пусть автомобиль, движется по горизонтальной прямой и, при этом увеличивает свою скорость (рис. 2). Масса автомобиля 1000 кг. В начале его скорость равнялась 1 м/с. После разгона скорость автомобиля равна 10 метрам в секунду. Найдем работу, которую пришлось проделать, чтобы ускорить этот автомобиль.

Для этого посчитаем энергию движения автомобиля в начале и в конце разгона.

( E_ left(text <дж>right) ) – начальная кинетическая энергия машины;

( E_ left(text <дж>right) ) – конечная кинетическая энергия машины;

( m left( text<кг>right) ) – масса автомобиля;

( displaystyle v left( frac<text<м>> right) ) – скорость, с которой машина движется.

Кинетическую энергию будем вычислять, используя формулу:

[ large E_ = 1000 cdot frac<1^<2>> <2>= 500 left(text <дж>right) ]

[ large E_ = 1000 cdot frac<10^<2>> <2>= 50000 left(text <дж>right) ]

Теперь найдем разницу кинетической энергии в конце и вначале разгона.

[ large Delta E_ = E_ — E_ ]

[ large Delta E_ = 50000 – 500 = 49500 left(text <дж>right) ]

Значит, работа, которую потребовалось совершить, чтобы разогнать машину массой 1000 кг от скорости 1 м/с до скорости 10 м/с, равняется 49500 Джоулям.

Примечание: Работа – это разность энергии в конце процесса и в его начале. Можно находить разность кинетической энергии, а можно — разность энергии потенциальной.

Работа силы тяжести — разность потенциальной энергии

Рассмотрим теперь следующий пример. Яблоко массой 0,2 кг упало на садовый стол с ветки, находящейся на высоте 3 метра от поверхности земли. Столешница располагается на высоте 1 метр от поверхности (рис. 3). Найдем работу силы тяжести в этом процессе.

Посчитаем потенциальную энергию яблока до его падения и энергию яблока на столешнице.

( E_ left(text <дж>right) ) – начальная потенциальная энергия яблока;

( E_ left(text <дж>right) ) – конечная потенциальная энергия яблока;

Примечание: Работу можно рассчитать через разность потенциальной энергии тела.

Потенциальную энергию будем вычислять, используя формулу:

[ large E_

= m cdot g cdot h]

( m left( text<кг>right) ) – масса яблока;

( h left( text<м>right) ) – высота, на которой находится яблоко относительно поверхности земли.

Начальная высота яблока над поверхностью земли равна 3 метрам

[ large E_ = 0,2 cdot 10 cdot 3 = 6 left(text <дж>right) ]

Потенциальная энергия яблока на столе

[ large E_ = 0,2 cdot 10 cdot 1 = 2 left(text <дж>right) ]

Теперь найдем разницу потенциальной энергии яблока в конце падения и перед его началом.

[ large Delta E_

= E_ — E_ ]

[ large Delta E_

= 2 – 6 = — 4 left(text <дж>right) ]

Важно помнить: Когда тело падает на землю, его потенциальная энергия уменьшается. Сила тяжести при этом совершает положительную работу!

Чтобы работа получилась положительной, в правой части формулы перед ( Delta E_

) дополнительно допишем знак «минус».

Читайте также:  Как измерить массу пустого стакана

Значит, работа, которую потребовалось совершить силе тяжести, чтобы яблоко массой 0,2 кг упало с высоты 3 м на высоту 1 метр, равняется 4 Джоулям.

Примечания:

  1. Если тело падает на землю, работа силы тяжести положительна;
  2. Когда мы поднимаем тело над землей, мы совершаем работу против силы тяжести. Наша работа при этом положительна, а работа силы тяжести будет отрицательной;
  3. Сила тяжести относится к консервативным силам. Для консервативных сил перед разностью потенциальной энергии мы дописываем знак «минус»;
  4. Работа силы тяжести не зависит от траектории, по которой двигалось тело;
  5. Работа для силы (displaystyle F_<text<тяж>>) зависит только от разности высот, в которых тело находилось в конечный и начальный моменты времени.

Рисунок 4 иллюстрирует факт, что для силы (displaystyle F_<text<тяж>>) работа зависит только от разности высот и не зависит от траектории, по которой тело двигалось.

Мощность

В механике мощность часто обозначают символами N или P и измеряют в Ваттах в честь шотландского изобретателя Джеймса Уатта.

Примечание: Символ (vec) используется для обозначения силы реакции опоры — она измеряется в Ньютонах и является векторной величиной. Чтобы не возникло путаницы, мощность вместо N будем обозначать символом P. Символ P – первая буква в английском слове power – мощность.

Мощность – это работа, совершенная за одну секунду (энергия, затраченная за 1 сек).

Расчет работы осуществляем, используя любую из формул:

[ large A = Delta E_ ]

[ large A = Delta E_

]

[ large A = F cdot S cdot cos(alpha) ]

Разделив эту работу на время, в течение которого она совершалась, получим мощность.

Если работа совершалась равными частями за одинаковые интервалы времени – мощность будет постоянной величиной.

Мощность переменная, когда в некоторые интервалы времени совершалось больше работы.

Еще одна формула для расчета мощности

Есть еще один способ расчета мощности, когда сила перемещает тело и при этом скорость тела не меняется:

[ large P = left( vec , vec right) ]

Формулу можно записать в скалярном виде:

[ large P = left| vec right| cdot left| vec right| cdot cos(alpha) ]

( F left( H right) ) – сила, перемещающая тело;

( displaystyle v left( frac<text<м>> right) ) – скорость тела;

( alpha ) – угол между вектором силы и вектором скорости тела;

Когда векторы (vec) и (vec) параллельны, запись формулы упрощается:

Примечание: Такую формулу для расчета мощности можно получить из выражения для работы силы, разделив обе части этого выражения на время, в течение которого работа совершалась (а если точнее, найдя производную обеих частей уравнения).

КПД – коэффициент полезного действия. Обычно обозначают греческим символом (eta) «эта». Единиц измерения не имеет, выражается либо десятичной дробью, либо в процентах.

Примечания:

  1. Процент – это дробь, у которой в знаменателе число 100.
  2. КПД — это либо правильная дробь, или дробь, равная единице.

Вычисляют коэффициент (eta) для какого-либо устройства, механизма или процесса.

( large A_<text<полезная>> left(text <дж>right)) – полезная работа;

(large A_<text<вся>> left(text <дж>right)) – вся затраченная для выполнения работы энергия;

Примечание: КПД часто меньше единицы, так как всегда есть потери энергии. Коэффициент полезного действия не может быть больше единицы, так как это противоречит закону сохранения энергии.

Величина (eta) является дробной величиной. Если числитель и знаменатель дроби разделить на одно и то же число, полученная дробь будет равна исходной. Используя этот факт, можно вычислять КПД, используя мощности:

Механическая работа — определение, основные формулы и примеры вычислений

Механическая работа – это одна из основных скалярных величин в физике. В рамках стандартной школьной программы она изучается в седьмом классе в разделе механики. Механическая работа – один из способов изменения внутренней энергии тела или субстанции (например, газа или жидкости) наряду с такими формами теплопередачи, как теплопроводность, конвекция и излучение, которые изучаются в разделе тепловых явлений.

Что такое работа в физике – определение и формула

Механическая работа – это количество энергии, которое нужно затратить для того, чтобы тело начало равномерно замедляющееся движение и прошло некоторую дистанцию.

В физике механической работой называется произведение силы, которая действует на некоторое тело, на расстояние, которое оно проходит под ее воздействием:

В более сложных случаях в формуле появляется и третья величина – косинус угла, под которым друг к другу расположены векторы движения и приложенной силы. Найти ее значение можно по формуле:

Для чего нужен расчет КПД

Коэффициент полезного действия электрической цепи – это отношение полезного тепла к полному. Для ясности приведем пример. При нахождении КПД двигателя можно определить, оправдывает ли его основная функция работы затраты потребляемого электричества. То есть его расчет даст ясную картину, насколько хорошо устройство преобразовывает получаемую энергию. Обратите внимание! Как правило, коэффициент полезного действия не имеет величины, а представляет собой процентное соотношение либо числовой эквивалент от 0 до 1. КПД находят по общей формуле вычисления, для всех устройств в целом. Но чтобы получить его результат в электрической цепи, вначале потребуется найти силу электричества.

Будет интересно➡ Что такое плотность тока?

По физике известно, что любой генератор тока имеет свое сопротивление, которое еще принято называть внутренняя мощность. Помимо этого значения, источник электричества также имеет свою силу. Дадим значения каждому элементу цепи: сопротивление – r; сила тока – Е; резистор (внешняя нагрузка) – R. Полная цепь Итак, чтобы найти силу тока, обозначение которого будет – I, и напряжение на резисторе – U, потребуется время – t, с прохождением заряда q = lt. Рассчитать работу источника тока можно по следующей формуле: A = Eq = EIt. В связи с тем, что сила электричества постоянна, работа генератора целиком преобразуется в тепло, выделяемое на R и r. Такое количество можно рассчитать по закону Джоуля-Ленца: Q = I2 + I2 rt = I2 (R + r) t.

Формулы расчета КПД.

Затем приравниваются правые части формулы: EIt = I2 (R + r) t. Осуществив сокращение, получается расчет: E = I(R + r). Произведя у формулы перестановку, в итоге получается: I = E R + r. Данное итоговое значение будет являться электрической силой в данном устройстве. Произведя таким образом предварительный расчет, теперь можно определить КПД.

Расчет КПД электрической цепи Мощность, получаемая от источника тока, называется потребляемой, определение ее записывается – P1. Если эта физическая величина переходит от генератора в полную цепь, она считается полезной и записывается – Р2. Чтобы определить КПД цепи, необходимо вспомнить закон сохранения энергии.

В соответствии с ним, мощность приемника Р2 будет всегда меньше потребляемой мощности Р1. Это объясняется тем, что в процессе работы в приемнике всегда происходит неизбежная пустая трата преобразуемой энергии, которая расходуется на нагревание проводов, их оболочки, вихревых токов и т.д. Чтобы найти оценку свойств превращения энергии, необходим КПД, который будет равен отношению мощностей Р2 и Р1.

Итак, зная все значения показателей, составляющих электроцепи, находим ее полезную и полную работу: А полезная. = qU = IUt =I2Rt; А полная = qE = IEt = I2(R+r)t. В соответствии этих значений, найдем мощности источника тока: Р2 = А полезная /t = IU = I2 R; P1 = А полная /t = IE = I2 (R + r). Произведя все действия, получаем формулу КПД: n = А полезная / А полная = Р2 / P1 =U / E = R / (R +r). У этой формулы получается, что R выше бесконечности, а n выше 1, но при всем этом ток в цепи остается в низком положении, и его полезная мощность мала.

Каждый желает найти КПД повышенного значения. Для этого необходимо найти условия, при которых P2 будет максимален. Оптимальные значения будут: dP2 / dR = 0. Далее определить КПД можно формулами: P2 = I2 R = (E / R + r)2 R; dP2 / dR = (E2 (R + r)2 — 2 (r + R) E2 R) / (R + r)4 = 0; E2 ((R + r) -2R) = 0. В данном выражении Е и (R + r) не равны 0, следовательно, ему равно выражение в скобках, то есть (r = R). Тогда получается, что мощность имеет максимальное значение, а коэффициент полезного действия = 50 %. Как видно, найти коэффициент полезного действия электрической цепи можно самостоятельно, не прибегая к услугам специалиста. Главное –соблюдать последовательность в расчетах и не выходить за рамки приведенных формул.

Примеры расчета КПД

Пример 1. Нужно рассчитать коэффициент для классического камина. Дано: удельная теплота сгорания березовых дров – 107Дж/кг, количество дров – 8 кг. После сгорания дров температура в комнате повысилась на 20 градусов. Удельная теплоемкость кубометра воздуха – 1,3 кДж/ кг*град. Общая кубатура комнаты – 75 кубометров.

Чтобы решить задачу, нужно найти частное или отношение двух величин. В числителе будет количество теплоты, которое получил воздух в комнате (1300Дж*75*20=1950 кДж ). В знаменателе – количество теплоты, выделенное дровами при горении (10000000Дж*8 =8*107 кДж). После подсчетов получаем, что энергоэффективность дровяного камина – около 2,5%. Действительно, современная теория об устройстве печей и каминов говорит, что классическая конструкция не является энергоэффективной. Это связано с тем, что труба напрямую выводит горячий воздух в атмосферу.

Для повышения эффективности устраивают дымоход с каналами, где воздух сначала отдает тепло кладке каналов, и лишь потом выходит наружу. Но справедливости ради, нужно отметить, что в процессе горения камина нагревается не только воздух, но и предметы в комнате, а часть тепла выходит наружу через элементы, плохо теплоизолированные – окна, двери и т.д.

Расчет коэффициента полезного действия.

Пример 2. Автомобиль проделал путь 100 км. Вес машины с пассажирами и багажом – 1400 кг. При этом было затрачено14 литров бензина. Найти: КПД двигателя.

Для решения задачи необходимо отношение работы по перемещению груза к количеству тепла, выделившемуся при сгорании топлива. Количество тепла также измеряется в Джоулях, поэтому не придется приводить к другим единицам. A будет равна произведению силы на путь( A=F*S=m*g*S). Сила равна произведению массы на ускорение свободного падения. Полезная работа = 1400 кг x 9,8м/с2 x 100000м=1,37*108 Дж

Удельная теплота сгорания бензина – 46 МДж/кг=46000 кДж/кг. Восемь литров бензина будем считать примерно равными 8 кг. Тепла выделилось 46*106*14=6.44*108 Дж. В результате получаем η ≈21%.

Часто задаваемые вопросы

Почему коэффициент полезного действия всегда меньше 100%?

КПД 100% означает, что вся энергия, затраченная на получение мощности двигателя, используется им в работе. В природе такого, в принципе, никогда не бывает, и поэтому КПД всех двигателей всегда меньше 100 процентов.

Как повысить коэффициент полезного действия механизма?

КПД механизмов можно увеличить, снижая трение в подвижных узлах и вес всех составных элементов конструкции. Для этого нужны новые смазочные вещества и лёгкие, но прочные конструкционные материалы.

Чему равен коэффициент полезного действия неподвижного блока?

Например, поднимая груз с помощью подвижного блока, приходится вместе с грузом поднимать и блок, а при этом необходимо совершать «дополнительную» работу. Отношение полезной работы Апол к совершенной Асов, выраженное в процентах, обозначают η и называют коэффициентом полезного действия (КПД): η = Апол/Асов · 100%.

Мощность разных устройств

По статистике, во время работы прибора теряется до 25% энергии. При функционировании двигателя внутреннего сгорания топливо сгорает частично. Небольшой процент вылетает в выхлопную трубу. При запуске бензиновый мотор греет себя и составные элементы. На потерю уходит до 35% от общей мощности.

При движении механизмов происходит трение. Для его ослабления используется смазка. Но она неспособна полностью устранить явление, поэтому затрачивается до 20% энергии. Пример на автомобиле: если расход составляет 10 литров топлива на 100 км, на движение потребуется 2 л, а остаток, равный 8 л — потеря.

Если сравнивать КПД бензинового и дизельного моторов, полезная мощность первого механизма равна 25%, а второго — 40%. Агрегаты схожи между собой, но у них разные виды смесеобразования:

  1. Поршни бензинового мотора функционируют на высоких температурах, поэтому нуждаются в хорошем охлаждении. Тепло, которое могло бы перейти в механическую энергию, тратится впустую, что способствует снижению КПД.
  2. В цепи дизельного устройства топливо воспламеняется в процессе сжатия. На основе данного фактора можно сделать вывод, что давление в цилиндрах высокое, при этом мотор экологичнее и меньше первого аналога. Если проверить КПД при низком функционировании и большом объёме, результат превысит 50%.

Асинхронные механизмы

Расшифровка термина «асинхронность» — несовпадение по времени. Понятие используется во многих современных машинах, которые являются электрическими и способны преобразовывать соответствующую энергию в механическую. Плюсы устройств:

  • простое изготовление;
  • низкая цена;
  • надёжность;
  • незначительные эксплуатационные затраты.

Чтобы рассчитать КПД, используется уравнение η = P2 / P1. Для расчёта Р1 и Р2 применяются общие данные потери энергии в обмотках мотора. У большинства агрегатов показатель находится в пределах 80−90%. Для быстрого расчёта используется онлайн-ресурс либо личный калькулятор. Для проверки возможного КПД у мотора внешнего сгорания, который функционирует от разных источников тепла, используется силовой агрегат Стирлинга. Он представлен в виде тепловой машины с рабочим телом в виде жидкости либо газа. Вещество движется по замкнутому объёму.

Принцип его функционирования основан на постепенном нагреве и охлаждении объекта за счёт извлечения энергии из давления. Подобный механизм применяется на косметическом аппарате и современной подводной лодке. Его работоспособность наблюдается при любой температуре. Он не нуждается в дополнительной системе для запуска. Его КПД возможно расширить до 70%, в отличие от стандартного мотора.

Значения показателя

Инженер Карно дал определение КПД

В 1824 году инженер Карно дал определение КПД идеального двигателя, когда коэффициент равен 100%. Для трактовки понятия была создана специальная машина со следующей формулой: η=(T1 — Т2)/ T1. Для расчёта максимального показателя применяется уравнение КПД макс = (T1-T2)/T1x100%. В двух примерах T1 указывает на температуру нагревателя, а T2 — температуру холодильника.

На практике для достижения 100% коэффициента потребуется приравнять температуру охладителя к нулю. Подобное явление невозможно, так как T1 выше температуры воздуха. Процедура повышения КПД источника тока либо силового агрегата считается важной технической задачей. Теоретически проблема решается путём снижения трения элементов двигателя и уменьшения теплопотери. В дизельном моторе подобное достигается турбонаддувом. В таком случае КПД возрастает до 50%.

Мощность стандартного двигателя увеличивается следующими способами:

  • подключение к системе многоцилиндрового агрегата;
  • применение специального топлива;
  • замена некоторых деталей;
  • перенос места сжигания бензина.

КПД зависит от типа и конструкции мотора. Современные учёные утверждают, что будущее за электродвигателями. На практике работа, которую совершает любое устройство, превышает полезную, так как определённая её часть выполняется против трения. Если используется подвижный блок, совершается дополнительная работа: поднимается блок с верёвкой, преодолеваются силы трения в блоке.

Общий коэффициент полезного действия, формула

При многократном превращении или передаче энергии общий коэффициент полезного действия равен произведению КПД на всех ступенях преобразования энергии:

[η_{общ} = η_1 · η_2 · η_3 … η_m ]

Примеры расчета КПД

Пример 1. Нужно рассчитать коэффициент для классического камина. Дано: удельная теплота сгорания березовых дров – 107Дж/кг, количество дров – 8 кг. После сгорания дров температура в комнате повысилась на 20 градусов. Удельная теплоемкость кубометра воздуха – 1,3 кДж/ кг*град. Общая кубатура комнаты – 75 кубометров.

Чтобы решить задачу, нужно найти частное или отношение двух величин. В числителе будет количество теплоты, которое получил воздух в комнате (1300Дж*75*20=1950 кДж ). В знаменателе – количество теплоты, выделенное дровами при горении (10000000Дж*8 =8*107 кДж). После подсчетов получаем, что энергоэффективность дровяного камина – около 2,5%. Действительно, современная теория об устройстве печей и каминов говорит, что классическая конструкция не является энергоэффективной. Это связано с тем, что труба напрямую выводит горячий воздух в атмосферу. Для повышения эффективности устраивают дымоход с каналами, где воздух сначала отдает тепло кладке каналов, и лишь потом выходит наружу. Но справедливости ради, нужно отметить, что в процессе горения камина нагревается не только воздух, но и предметы в комнате, а часть тепла выходит наружу через элементы, плохо теплоизолированные – окна, двери и т.д.

формула

Пример 2. Автомобиль проделал путь 100 км. Вес машины с пассажирами и багажом – 1400 кг. При этом было затрачено14 литров бензина. Найти: КПД двигателя.

Для решения задачи необходимо отношение работы по перемещению груза к количеству тепла, выделившемуся при сгорании топлива. Количество тепла также измеряется в Джоулях, поэтому не придется приводить к другим единицам. A будет равна произведению силы на путь( A=F*S=m*g*S). Сила равна произведению массы на ускорение свободного падения. Полезная работа = 1400 кг x 9,8м/с2 x 100000м=1,37*108 Дж

Удельная теплота сгорания бензина – 46 МДж/кг=46000 кДж/кг. Восемь литров бензина будем считать примерно равными 8 кг. Тепла выделилось 46*106*14=6.44*108 Дж. В результате получаем η ≈21%.

Единицы измерения

Коэффициент полезного действия – величина безразмерная, то есть не нужно ставить какую-либо единицу измерения. Но эту величину можно выразить и в процентах. Для этого полученное в результате деления по формуле число необходимо умножить на 100%. В школьном курсе математики рассказывали, что процент – этот одна сотая чего-либо. Умножая на 100 процентов, мы показываем, сколько в числе сотых.

КПД механизма — по какой формуле вычисляют

Человек придумал разнообразные механизмы, с помощью которых можно поднимать тяжёлые грузы на определённую высоту. Так, для подъёма ведра с водой из колодца изобрели ворот, для подъёма автомобиля – домкрат. При помощи лебёдки и наклонной плоскости египтяне построили свои грандиозные пирамиды.

Пользуясь этими приспособлениями, человек редко вспоминает об их КПД. В качестве примера рассмотрим этот показатель для наклонной плоскости.

Груз

Принцип расчёта КПД остаётся неизменным: нужно найти отношение полезной работы ко всей затраченной энергии. То есть опять-таки используем общую формулу (1), сделав соответствующие преобразования.

Предположим, тело массой m нужно поднять (точнее затолкать или затянуть) на высоту h. При постоянной скорости подъёма полезная работа будет равна произведению силы тяжести (mg) на высоту (h).

Затраченная работа определяется произволением силы толчка или тяги F на длину наклонной плоскости L. Заметим, что толчковое (тяговое) усилие идёт на преодоление силы трения Fтр.

Таким образом, КПД такого простейшего механизма можно посчитать по формуле:

Сила трения

Несложный анализ показывает, что КПД наклонной плоскости обратно пропорционален силе трения и длине аппарели. Последняя, в свою очередь, зависит от угла наклона: чем он больше, тем короче аппарель.

Физическая формула КПД

С учетом изложенных выше особенностей и необходимости выражения результата в %, физические формулы приобретают усовершенствованный внешний вид:

Физическая формула КПД

или

Физическая формула КПД 2

Как обозначается

В русских учебниках обозначается двояко. Либо так и пишется – КПД, либо обозначается греческой буквой η. Эти обозначения равнозначны.

Решение примеров

Задача 1. Поезд на скорости 54 км/ч развивает мощность 720 кВт. Нужно вычислить силу тяги силовых агрегатов. Решение: чтобы найти мощность, используется формула N=F x v. Если перевести скорость в единицу СИ, получится 15 м/с. Подставив данные в уравнение, определяется, что F равно 48 kН.

Задача 2. Масса транспортного средства соответствует 2200 кг. Машина, поднимаясь в гору под уклоном в 0,018, проходит расстояние 100 м. Скорость развивается до 32,4 км/ч, а коэффициент трения соответствует 0,04. Нужно определить среднюю мощность авто при движении. Решение: вычисляется средняя скорость — v/2. Чтобы определить силу тяги мотора, выполняется рисунок, на котором отображаются силы, воздействующие на машину:

  • тяжесть — mg;
  • реакция опоры — N;
  • трение — Ftr;
  • тяга — F.

Первая величина вычисляется по второму закону Ньютона: mg+N+Ftr+F=ma. Для ускорения используется уравнение a=v2/2S. Если подставить последние значение и воспользоваться cos, получится средняя мощность. Так как ускорение считается постоянной величиной и равно 9,8 м/с2, поэтому v= 9 м/с. Подставив данные в первую формулу, получится: N= 9,5 kBt.

При решении сложных задач по физике рекомендуется проверить соответствие предоставленных в условиях единиц измерения с международными стандартами. Если они отличаются, необходимости перевести данные с учётом СИ.

Видео

Это видео поможет вам понять, что такое КПД.

Мощность и КПД

Мощность механизма или устройства равна работе, совершаемой в единицу времени. Работа(A) измеряется в Джоулях, а время в системе Си – в секундах. Но не стоит путать понятие мощности и номинальной мощности. Если на чайнике написана мощность 1 700 Ватт, это не значит, что он передаст 1 700 Джоулей за одну секунду воде, налитой в него. Это мощность номинальная. Чтобы узнать η электрочайника, нужно узнать количество теплоты(Q), которое должно получить определенное количество воды при нагреве на энное количество градусов. Эту цифру делят на работу электрического тока, выполненную за время нагревания воды.

Величина A будет равна номинальной мощности, умноженной на время в секундах. Q будет равно объему воды, умноженному на разницу температур на удельную теплоемкость. Потом делим Q на A тока и получаем КПД электрочайника, примерно равное 80 процентам. Прогресс не стоит на месте, и КПД различных устройств повышается, в том числе бытовой техники.

определение

Напрашивается вопрос, почему через мощность нельзя узнать КПД устройства. На упаковке с оборудованием всегда указана номинальная мощность. Она показывает, сколько энергии потребляет устройство из сети. Но в каждом конкретном случае невозможно будет предсказать, сколько конкретно потребуется энергии для нагрева даже одного литра воды.

Например, в холодной комнате часть энергии потратится на обогрев пространства. Это связано с тем, что в результате теплообмена чайник будет охлаждаться. Если, наоборот, в комнате будет жарко, чайник закипит быстрее. То есть КПД в каждом из этих случаев будет разным.

Полезная и затраченная работа

Был такой мифологический персонаж у древних греков — Сизиф. За то, что он обманул богов, те приговорили его после смерти вечно таскать огромный булыжник вверх по горе, откуда этот булыжник скатывался — и так без конца. В общем, Сизиф делал совершенно бесполезное дело с нулевым КПД. Поэтому бесполезную работу и называют «сизифов труд».

Чтобы разобраться в понятиях полезной и затраченной работы, давайте пофантазируем и представим, что Сизифа помиловали и камень больше не скатывается с горы, а КПД перестал быть нулевым.

Полезная работа в этом случае равна потенциальной энергии, приобретенной булыжником. Потенциальная энергия, в свою очередь, прямо пропорциональна высоте: чем выше расположено тело, тем больше его потенциальная энергия. Выходит, чем выше Сизиф прикатил камень, тем больше полезная работа.

Потенциальная энергия

Еп = mgh

m — масса тела [кг]

g — ускорение свободного падения [м/с2]

h — высота [м]

На планете Земля g ≈ 9,8 м/с2

Затраченная работа в нашем примере — это механическая работа Сизифа. Механическая работа зависит от приложенной силы и пути, на протяжении которого эта сила была приложена.

Механическая работа

А = FS

A — механическая работа [Дж]

F — приложенная сила [Н]

S — путь [м]

И как же достоверно определить, какая работа полезная, а какая затраченная?

Все очень просто! Задаем два вопроса:

  1. За счет чего происходит процесс?
  2. Ради какого результата?

В примере выше процесс происходит ради того, чтобы тело поднялось на какую-то высоту, а значит — приобрело потенциальную энергию (для физики это синонимы).

Происходит процесс за счет энергии, затраченной Сизифом — вот и затраченная работа.

Мощность

На заводах по всему миру большинство задач выполняют машины. Например, если нам нужно закрыть крышечками тысячу банок колы, аппарат сделает это в считанные минуты. У человека эта задача заняла бы намного больше времени. Получается, что машина и человек выполняют одинаковую работу за разные промежутки времени. Для того, чтобы описать скорость выполнения работы, нам потребуется понятие мощности.

Мощностью называется физическая величина, равная отношению работы ко времени ее выполнения.

Мощность

N = A/t

N — мощность [Вт]

A — механическая работа [Дж]

t — время [с]

Один ватт — это мощность, при которой работа в один джоуль совершается за одну секунду.

Также для мощности справедлива другая формула:

Мощность

N = Fv

N — мощность [Вт]

F — приложенная сила [Н]

v — скорость [м/с]

Как и для работы, для мощности справедливо правило знаков: если векторы направлены противоположно, значение мощности будет отрицательным.

Поскольку сила и скорость — векторные величины, в случае наличия между ними угла формула принимает следующий вид:

Мощность

N = Fvcosα

N — мощность [Вт]

F — приложенная сила [Н]

v — скорость [м/с]

α — угол между векторами силы и скорости []

Оглавление:

  • Основные теоретические сведения
    • Механическая работа
    • Мощность
    • Кинетическая энергия
    • Потенциальная энергия
    • Коэффициент полезного действия
    • Закон сохранения механической энергии
    • Некоторые рекомендации к решению задач на работу
    • Закон сохранения энергии и динамика вращательного движения
    • Неупругие соударения
    • Абсолютно упругий удар
    • Рекомендации к решению некоторых сложных задач на законы сохранения

Основные теоретические сведения

Механическая работа

К оглавлению…

Энергетические характеристики движения вводятся на основе понятия механической работы или работы силы. Работой, совершаемой постоянной силой F, называется физическая величина, равная произведению модулей силы и перемещения, умноженному на косинус угла между векторами силы F и перемещения S:

Формула Механическая работа

Работа является скалярной величиной. Она может быть как положительна (0° ≤ α < 90°), так и отрицательна (90° < α ≤ 180°). При α = 90° работа, совершаемая силой, равна нулю. В системе СИ работа измеряется в джоулях (Дж). Джоуль равен работе, совершаемой силой в 1 ньютон на перемещении 1 метр в направлении действия силы.

Если же сила изменяется с течением времени, то для нахождения работы строят график зависимости силы от перемещения и находят площадь фигуры под графиком – это и есть работа:

Работа как площадь под графиком

Примером силы, модуль которой зависит от координаты (перемещения), может служить сила упругости пружины, подчиняющаяся закону Гука (Fупр = kx).

Мощность

К оглавлению…

Работа силы, совершаемая в единицу времени, называется мощностью. Мощность P (иногда обозначают буквой N) – физическая величина, равная отношению работы A к промежутку времени t, в течение которого совершена эта работа:

Формула Мощность

По этой формуле рассчитывается средняя мощность, т.е. мощность обобщенно характеризующая процесс. Итак, работу можно выражать и через мощность: A = Pt (если конечно известна мощность и время совершения работы). Единица мощности называется ватт (Вт) или 1 джоуль за 1 секунду. Если движение равномерное, то:

Формула Мгновенная механическая мощность

По этой формуле мы можем рассчитать мгновенную мощность (мощность в данный момент времени), если вместо скорости подставим в формулу значение мгновенной скорости. Как узнать, какую мощность считать? Если в задаче спрашивают мощность в момент времени или в какой-то точке пространства, то считается мгновенная. Если спрашивают про мощность за какой-то промежуток времени или участок пути, то ищите среднюю мощность.

КПД – коэффициент полезного действия, равен отношению полезной работы к затраченной, либо же полезной мощности к затраченной:

Формула Коэффициент полезного действия (КПД)

Какая работа полезная, а какая затраченная определяется из условия конкретной задачи путем логического рассуждения. К примеру, если подъемный кран совершает работу по подъему груза на некоторую высоту, то полезной будет работа по поднятию груза (так как именно ради нее создан кран), а затраченной – работа, совершенная электродвигателем крана.

Итак, полезная и затраченная мощность не имеют строгого определения, и находятся логическим рассуждением. В каждой задаче мы сами должны определить, что в этой задаче было целью совершения работы (полезная работа или мощность), а что было механизмом или способом совершения всей работы (затраченная мощность или работа).

В общем случае КПД показывает, как эффективно механизм преобразует один вид энергии в другой. Если мощность со временем изменяется, то работу находят как площадь фигуры под графиком зависимости мощности от времени:

Работа как площадь под графиком

Кинетическая энергия

К оглавлению…

Физическая величина, равная половине произведения массы тела на квадрат его скорости, называется кинетической энергией тела (энергией движения):

Формула для кинетической энергии

То есть если автомобиль массой 2000 кг движется со скоростью 10 м/с, то он обладает кинетической энергией равной Ек = 100 кДж и способен совершить работу в 100 кДж. Эта энергия может превратиться в тепловую (при торможении автомобиля нагревается резина колес, дорога и тормозные диски) или может быть потрачена на деформацию автомобиля и тела, с которым автомобиль столкнулся (при аварии). При вычислении кинетической энергии не имеет значения куда движется автомобиль, так как энергия, как и работа, величина скалярная.

Тело обладает энергией, если способно совершить работу. Например, движущееся тело обладает кинетической энергией, т.е. энергией движения, и способно совершать работу по деформации тел или придания ускорения телам, с которыми произойдёт столкновение.

Физический смысл кинетической энергии: для того чтобы покоящееся тело массой m стало двигаться со скоростью v необходимо совершить работу равную полученному значению кинетической энергии. Если тело массой m движется со скоростью v, то для его остановки необходимо совершить работу равную его первоначальной кинетической энергии. При торможении кинетическая энергия в основном (кроме случаев соударения, когда энергия идет на деформации) «забирается» силой трения.

Теорема о кинетической энергии: работа равнодействующей силы равна изменению кинетической энергии тела:

Теорема о кинетической энергии

Теорема о кинетической энергии справедлива и в общем случае, когда тело движется под действием изменяющейся силы, направление которой не совпадает с направлением перемещения. Применять данную теорему удобно в задачах на разгон и торможение тела.

Потенциальная энергия

К оглавлению…

Наряду с кинетической энергией или энергией движения в физике важную роль играет понятие потенциальной энергии или энергии взаимодействия тел.

Потенциальная энергия определяется взаимным положением тел (например, положением тела относительно поверхности Земли). Понятие потенциальной энергии можно ввести только для сил, работа которых не зависит от траектории движения тела и определяется только начальным и конечным положениями (так называемые консервативные силы). Работа таких сил на замкнутой траектории равна нулю. Таким свойством обладают сила тяжести и сила упругости. Для этих сил можно ввести понятие потенциальной энергии.

Потенциальная энергия тела в поле силы тяжести Земли рассчитывается по формуле:

Формула Потенциальная энергия тела поднятого на высоту

Физический смысл потенциальной энергии тела: потенциальная энергия равна работе, которую совершает сила тяжести при опускании тела на нулевой уровень (h – расстояние от центра тяжести тела до нулевого уровня). Если тело обладает потенциальной энергией, значит оно способно совершить работу при падении этого тела с высоты h до нулевого уровня. Работа силы тяжести равна изменению потенциальной энергии тела, взятому с противоположным знаком:

Работа силы тяжести

Часто в задачах на энергию приходится находить работу по поднятию (переворачиванию, доставанию из ямы) тела. Во всех этих случаях нужно рассматривать перемещение не самого тела, а только его центра тяжести.

Потенциальная энергия Ep зависит от выбора нулевого уровня, то есть от выбора начала координат оси OY. В каждой задаче нулевой уровень выбирается из соображения удобства. Физический смысл имеет не сама потенциальная энергия, а ее изменение при перемещении тела из одного положения в другое. Это изменение не зависит от выбора нулевого уровня.

Потенциальная энергия растянутой пружины рассчитывается по формуле:

Формула Потенциальная энергия растянутой (или сжатой) пружины

где: k – жесткость пружины. Растянутая (или сжатая) пружина способна привести в движение прикрепленное к ней тело, то есть сообщить этому телу кинетическую энергию. Следовательно, такая пружина обладает запасом энергии. Растяжение или сжатие х надо рассчитывать от недеформированного состояния тела.

Потенциальная энергия упруго деформированного тела равна работе силы упругости при переходе из данного состояния в состояние с нулевой деформацией. Если в начальном состоянии пружина уже была деформирована, а ее удлинение было равно x1, тогда при переходе в новое состояние с удлинением x2 сила упругости совершит работу, равную изменению потенциальной энергии, взятому с противоположным знаком (так как сила упругости всегда направлена против деформации тела):

Работа силы упругости

Потенциальная энергия при упругой деформации – это энергия взаимодействия отдельных частей тела между собой силами упругости.

Работа силы трения зависит от пройденного пути (такой вид сил, чья работа зависит от траектории и пройденного пути называется: диссипативные силы). Понятие потенциальной энергии для силы трения вводить нельзя.

Коэффициент полезного действия

К оглавлению…

Коэффициент полезного действия (КПД) – характеристика эффективности системы (устройства, машины) в отношении преобразования или передачи энергии. Он определяется отношением полезно использованной энергии к суммарному количеству энергии, полученному системой (формула уже приведена выше).

КПД можно рассчитывать как через работу, так и через мощность. Полезная и затраченная работа (мощность) всегда определяются путем простых логических рассуждений.

В электрических двигателях КПД – отношение совершаемой (полезной) механической работы к электрической энергии, получаемой от источника. В тепловых двигателях – отношение полезной механической работы к затрачиваемому количеству теплоты. В электрических трансформаторах – отношение электромагнитной энергии, получаемой во вторичной обмотке, к энергии, потребляемой первичной обмоткой.

В силу своей общности понятие КПД позволяет сравнивать и оценивать с единой точки зрения такие различные системы, как атомные реакторы, электрические генераторы и двигатели, теплоэнергетические установки, полупроводниковые приборы, биологические объекты и т.д.

Из–за неизбежных потерь энергии на трение, на нагревание окружающих тел и т.п. КПД всегда меньше единицы. Соответственно этому КПД выражается в долях затрачиваемой энергии, то есть в виде правильной дроби или в процентах, и является безразмерной величиной. КПД характеризует как эффективно работает машина или механизм. КПД тепловых электростанций достигает 35–40%, двигателей внутреннего сгорания с наддувом и предварительным охлаждением – 40–50%, динамомашин и генераторов большой мощности – 95%, трансформаторов – 98%.

Задачу, в которой нужно найти КПД или он известен, надо начать с логического рассуждения – какая работа является полезной, а какая затраченной.

Закон сохранения механической энергии

К оглавлению…

Полной механической энергией называется сумма кинетической энергии (т.е. энергии движения) и потенциальной (т.е. энергии взаимодействия тел силами тяготения и упругости):

Формула Полная механическая энергия

Если механическая энергия не переходит в другие формы, например, во внутреннюю (тепловую) энергию, то сумма кинетической и потенциальной энергии остаётся неизменной. Если же механическая энергия переходит в тепловую, то изменение механической энергии равно работе силы трения или потерям энергии, или количеству выделившегося тепла и так далее, другими словами изменение полной механической энергии равно работе внешних сил:

Формула Связь полной механической энергии тела или системы тел и работы внешних сил

Сумма кинетической и потенциальной энергии тел, составляющих замкнутую систему (т.е. такую в которой не действует внешних сил, и их работа соответственно равна нолю) и взаимодействующих между собой силами тяготения и силами упругости, остается неизменной:

Формула Закон сохранения механической энергии (ЗСЭ)

Это утверждение выражает закон сохранения энергии (ЗСЭ) в механических процессах. Он является следствием законов Ньютона. Закон сохранения механической энергии выполняется только тогда, когда тела в замкнутой системе взаимодействуют между собой силами упругости и тяготения. Во всех задачах на закон сохранения энергии всегда будет как минимум два состояния системы тел. Закон гласит, что суммарная энергия первого состояния будет равна суммарной энергии второго состояния.

Алгоритм решения задач на закон сохранения энергии:

  1. Найти точки начального и конечного положения тела.
  2. Записать какой или какими энергиями обладает тело в данных точках.
  3. Приравнять начальную и конечную энергию тела.
  4. Добавить другие необходимые уравнения из предыдущих тем по физике.
  5. Решить полученное уравнение или систему уравнений математическими методами.

Важно отметить, что закон сохранения механической энергии позволил получить связь между координатами и скоростями тела в двух разных точках траектории без анализа закона движения тела во всех промежуточных точках. Применение закона сохранения механической энергии может в значительной степени упростить решение многих задач.

В реальных условиях практически всегда на движущиеся тела наряду с силами тяготения, силами упругости и другими силами действуют силы трения или силы сопротивления среды. Работа силы трения зависит от длины пути.

Если между телами, составляющими замкнутую систему, действуют силы трения, то механическая энергия не сохраняется. Часть механической энергии превращается во внутреннюю энергию тел (нагревание). Таким образом энергия в целом (т.е. не только механическая) в любом случае сохраняется.

При любых физических взаимодействиях энергия не возникает и не исчезает. Она лишь превращается из одной формы в другую. Этот экспериментально установленный факт выражает фундаментальный закон природы – закон сохранения и превращения энергии.

Одним из следствий закона сохранения и превращения энергии является утверждение о невозможности создания «вечного двигателя» (perpetuum mobile) – машины, которая могла бы неопределенно долго совершать работу, не расходуя при этом энергии.

Разные задачи на работу

К оглавлению…

Если в задаче требуется найти механическую работу, то сначала выберите способ её нахождения:

  1. Работу можно найти по формуле: A = FS∙cosα. Найдите силу, совершающую работу, и величину перемещения тела под действием этой силы в выбранной системе отсчёта. Обратите внимание, что угол должен быть выбран между векторами силы и перемещения.
  2. Работу внешней силы можно найти, как разность механической энергии в конечной и начальной ситуациях. Механическая энергия равна сумме кинетической и потенциальной энергий тела.
  3. Работу по подъёму тела с постоянной скоростью можно найти по формуле: A = mgh, где h – высота, на которую поднимается центр тяжести тела.
  4. Работу можно найти как произведение мощности на время, т.е. по формуле: A = Pt.
  5. Работу можно найти, как площадь фигуры под графиком зависимости силы от перемещения или мощности от времени.

Закон сохранения энергии и динамика вращательного движения

К оглавлению…

Задачи этой темы являются достаточно сложными математически, но при знании подхода решаются по совершенно стандартному алгоритму. Во всех задачах Вам придется рассматривать вращение тела в вертикальной плоскости. Решение будет сводиться к следующей последовательности действий:

  1. Надо определить интересующую Вас точку (ту точку, в которой необходимо определить скорость тела, силу натяжения нити, вес и так далее).
  2. Записать в этой точке второй закон Ньютона, учитывая, что тело вращается, то есть у него есть центростремительное ускорение.
  3. Записать закон сохранения механической энергии так, чтобы в нем присутствовала скорость тела в той самой интересной точке, а также характеристики состояния тела в каком-нибудь состоянии про которое что-то известно.
  4. В зависимости от условия выразить скорость в квадрате из одного уравнения и подставить в другое.
  5. Провести остальные необходимые математические операции для получения окончательного результата.

При решении задач надо помнить, что:

  • Условие прохождения верхней точки при вращении на нити с минимальной скоростью – сила реакции опоры N в верхней точке равна 0. Такое же условие выполняется при прохождении верхней точки мертвой петли.
  • При вращении на стержне условие прохождения всей окружности: минимальная скорость в верхней точке равна 0.
  • Условие отрыва тела от поверхности сферы – сила реакции опоры в точке отрыва равна нулю.

Неупругие соударения

К оглавлению…

Закон сохранения механической энергии и закон сохранения импульса позволяют находить решения механических задач в тех случаях, когда неизвестны действующие силы. Примером такого рода задач является ударное взаимодействие тел.

Ударом (или столкновением) принято называть кратковременное взаимодействие тел, в результате которого их скорости испытывают значительные изменения. Во время столкновения тел между ними действуют кратковременные ударные силы, величина которых, как правило, неизвестна. Поэтому нельзя рассматривать ударное взаимодействие непосредственно с помощью законов Ньютона. Применение законов сохранения энергии и импульса во многих случаях позволяет исключить из рассмотрения сам процесс столкновения и получить связь между скоростями тел до и после столкновения, минуя все промежуточные значения этих величин.

С ударным взаимодействием тел нередко приходится иметь дело в обыденной жизни, в технике и в физике (особенно в физике атома и элементарных частиц). В механике часто используются две модели ударного взаимодействия – абсолютно упругий и абсолютно неупругий удары.

Абсолютно неупругим ударом называют такое ударное взаимодействие, при котором тела соединяются (слипаются) друг с другом и движутся дальше как одно тело.

При абсолютно неупругом ударе механическая энергия не сохраняется. Она частично или полностью переходит во внутреннюю энергию тел (нагревание). Для описания любых ударов Вам нужно записать и закон сохранения импульса, и закон сохранения механической энергии с учетом выделяющейся теплоты (предварительно крайне желательно сделать рисунок).

Абсолютно упругий удар

К оглавлению…

Абсолютно упругим ударом называется столкновение, при котором сохраняется механическая энергия системы тел. Во многих случаях столкновения атомов, молекул и элементарных частиц подчиняются законам абсолютно упругого удара. При абсолютно упругом ударе наряду с законом сохранения импульса выполняется закон сохранения механической энергии. Простым примером абсолютно упругого столкновения может быть центральный удар двух бильярдных шаров, один из которых до столкновения находился в состоянии покоя.

Центральным ударом шаров называют соударение, при котором скорости шаров до и после удара направлены по линии центров. Таким образом, пользуясь законами сохранения механической энергии и импульса, можно определить скорости шаров после столкновения, если известны их скорости до столкновения. Центральный удар очень редко реализуется на практике, особенно если речь идет о столкновениях атомов или молекул. При нецентральном упругом соударении скорости частиц (шаров) до и после столкновения не направлены по одной прямой.

Частным случаем нецентрального упругого удара может служить соударения двух бильярдных шаров одинаковой массы, один из которых до соударения был неподвижен, а скорость второго была направлена не по линии центров шаров. В этом случае векторы скоростей шаров после упругого соударения всегда направлены перпендикулярно друг к другу.

Законы сохранения. Сложные задачи

К оглавлению…

Несколько тел

В некоторых задачах на закон сохранения энергии тросы с помощью которых перемещаются некие объекты могут иметь массу (т.е. не быть невесомыми, как Вы могли уже привыкнуть). В этом случае работу по перемещению таких тросов (а именно их центров тяжести) также нужно учитывать.

Если два тела, соединённые невесомым стержнем, вращаются в вертикальной плоскости, то:

  1. выбирают нулевой уровень для расчёта потенциальной энергии, например на уровне оси вращения или на уровне самой нижней точки нахождения одного из грузов и обязательно делают чертёж;
  2. записывают закон сохранения механической энергии, в котором в левой части записывают сумму кинетической и потенциальной энергии обоих тел в начальной ситуации, а в правой части записывают сумму кинетической и потенциальной энергии обоих тел в конечной ситуации;
  3. учитывают, что угловые скорости тел одинаковы, тогда линейные скорости тел пропорциональны радиусам вращения;
  4. при необходимости записывают второй закон Ньютона для каждого из тел в отдельности.

Разрыв снаряда

В случае разрыва снаряда выделяется энергия взрывчатых веществ. Чтобы найти эту энергию надо от суммы механических энергий осколков после взрыва отнять механическую энергию снаряда до взрыва. Также будем использовать закон сохранения импульса, записанный, в виде теоремы косинусов (векторный метод) или в виде проекций на выбранные оси.

Столкновения с тяжёлой плитой

Пусть навстречу тяжёлой плите, которая движется со скоростью v, движется лёгкий шарик массой m со скоростью uн. Так как импульс шарика много меньше импульса плиты, то после удара скорость плиты не изменится, и она будет продолжать движение с той же скоростью и в том же направлении. В результате упругого удара, шарик отлетит от плиты. Здесь важно понять, что не поменяется скорость шарика относительно плиты. В таком случае, для конечной скорости шарика получим:

Столкновение шарика и тяжёлой плиты

Таким образом, скорость шарика после удара увеличивается на удвоенную скорость стены. Аналогичное рассуждение для случая, когда до удара шарик и плита двигались в одном направлении, приводит к результату согласно которому скорость шарика уменьшается на удвоенную скорость стены:

Столкновение шарика и тяжёлой плиты

Задачи о максимальных и минимальных значениях энергии сталкивающихся шаров

В задачах такого типа главное понять, что потенциальная энергия упругой деформации шаров максимальна, если кинетическая энергия их движения минимальна – это следует из закона сохранения механической энергии. Сумма кинетических энергий шаров минимальна в тот момент, когда скорости шаров будут одинаковы по величине и направлены в одном направлении. В этот момент относительная скорость шаров равна нулю, а деформация и связанная с ней потенциальная энергия максимальна.

Содержание:

Работа, мощность и энергия:

Мы часто слышим от друзей: «Я сегодня выполнил большую работу: выучил наизусть стихотворение и решил пять задач по математике». Но с точки зрения физики никакой работы не совершено, даже если выучить наизусть целую поэму. Что же такое работа в физике?

В физике работа оценивает то, что вызвала сила, действуя на движущееся тело. Покажем это на примерах. Рассмотрите внимательно рисунок 216. Что общего в результатах действия силы тяжести на мяч (рис. 216, а), силы давления газа на пулю в пистолете (рис. 216, б) и силы упругости сжатой пружины на шарик (рис. 216, в) после пережигания нити? Все перечисленные силы вызывают разгон тел (мяча, пули, шарика), т. е. увеличение скорости движения.

Работа, мощность и энергия в физике - виды, формулы и определения с примерами

Л может ли сила, действующая на движущееся тело, уменьшать его скорость? Подбросьте мяч и наблюдайте за его движением вверх (рис. 217). Теперь сила тяжести уменьшает скорость его движения. Во всех случаях, когда сила изменяет скорость движения (увеличивает или уменьшает), говорят, что сила совершает механическую работу.

Работа, мощность и энергия в физике - виды, формулы и определения с примерами

Механическая работа является физической величиной. Ее значение можно рассчитать. Рассмотрим самый простой случай: направление силы совпадает с направлением движения. Например, идет разгон спортивных саней (рис. 218). Изменение скорости саней, а значит, и работа по их разгону зависят от значения действующей силы (силы спортсменов, разгоняющих сани) и от пройденного санями пути. Чем больше сила и путь, тем большая совершается работа. Этот вывод справедлив для всех движущихся под действием силы тел.

Работа, мощность и энергия в физике - виды, формулы и определения с примерами

Таким образом, механическая работа — физическая величина, пропорциональная действующей на тело силе и пройденному пути.

Обозначим работу буквой А. Тогда, если направление силы совпадает с направлением движения тела,

Работа, мощность и энергия в физике - виды, формулы и определения с примерами

Единицей работы в СИ является 1 джоуль (1 Дж). Названа она в честь известного английского физика Дж. П. Джоуля. Один джоуль — это работа, совершаемая силой 1 Н на пути 1 м.

1 джоуль = 1 ньютон • 1 метр.

Для измерения большой работы используют кратные джоулю единицы:

Работа, мощность и энергия в физике - виды, формулы и определения с примерами

В случае малой работы применяются дольные единицы:

Работа, мощность и энергия в физике - виды, формулы и определения с примерами

Из формулы работы следует, что если есть силы, но нет движения, то нет и работы. Например, сила тяжести, действующая на лежащий на столе мяч (рис. 219, а), работы не совершает, а в случае падающего мяча (рис. 219, б) — совершает.

Работа, мощность и энергия в физике - виды, формулы и определения с примерами

Сила не всегда увеличивает скорость движения тела. Так, при движении мяча вверх (см. рис. 217) сила тяжести замедляет его движение. Аналогично при скольжении шайбы по льду сила трения уменьшает скорость движения шайбы. Работу силы (тяжести, трения) в подобных случаях считают отрицательной.

Но положительная и отрицательная работы могут совершаться одновременно и даже быть равными по абсолютной величине. В этом случае скорость движения постоянна. Например, электропоезд на данном участке пути движется равномерно. Это значит, что равнодействующая сил (тяги двигателя и сопротивления движению) равна нулю. По и сила тяги, и сила сопротивления совершают работу. Только работа силы тяги Работа, мощность и энергия в физике - виды, формулы и определения с примерами а силы сопротивления Работа, мощность и энергия в физике - виды, формулы и определения с примерами Сумма же их равна 0, т. е. Работа, мощность и энергия в физике - виды, формулы и определения с примерами

Главные выводы:

  1. Механическая работа характеризует результат действия силы на движущееся тело и пропорциональна действующей на тело силе и пройденному телом пути.
  2. Силы, ускоряющие движение тела; совершают положительную работу.
  3. Силы, замедляющие движение тела, совершают отрицательную работу.
  4. Единица работы в СИ — 1 джоуль (1 Дж).

Пример решения задачи:

Подъемный кран равномерно поднимает с земли бетонную плиту массой m = 500 кг на один из этажей строящегося дома. Сила упругости троса при этом совершает работу А = 100 кДж. Определите, на какой этаж была поднята плита, если высота одного этажа Работа, мощность и энергия в физике - виды, формулы и определения с примерами Чему равна работа равнодействующей сил, приложенных к плите? Коэффициент Работа, мощность и энергия в физике - виды, формулы и определения с примерами примите равным Работа, мощность и энергия в физике - виды, формулы и определения с примерами

Дано:

Работа, мощность и энергия в физике - виды, формулы и определения с примерами

Работа, мощность и энергия в физике - виды, формулы и определения с примерами

Работа, мощность и энергия в физике - виды, формулы и определения с примерами

Работа, мощность и энергия в физике - виды, формулы и определения с примерами

Работа, мощность и энергия в физике - виды, формулы и определения с примерами

Решение:

При равномерном подъеме сила упругости троса равна силе тяжести, действующей на плиту: Работа, мощность и энергия в физике - виды, формулы и определения с примерами

Работа силы упругости Работа, мощность и энергия в физике - виды, формулы и определения с примерами Высота подъема Работа, мощность и энергия в физике - виды, формулы и определения с примерами — число этажей. Тогда Работа, мощность и энергия в физике - виды, формулы и определения с примерами

Отсюда  Работа, мощность и энергия в физике - виды, формулы и определения с примерами

Так как движение плиты равномерное, то равнодействующая сил, приложенных к ней, Работа, мощность и энергия в физике - виды, формулы и определения с примерами и работа Работа, мощность и энергия в физике - виды, формулы и определения с примерами

Ответ: плита поднята на 6-й этаж; работа равнодействующей сил Работа, мощность и энергия в физике - виды, формулы и определения с примерами

Полезная и совершённая работа

Оценивая работу машины, механизма и др., говорят об их коэффициенте полезного действия (КПД). Но что такое КПД? Что означают слова «полезного действия»? А что такое неполезное действие?

Рассмотрим ситуацию: идет уборка картофеля на поле. Фермер поднимает картофель в ведре в кузов автомашины (рис. 221), выгружает, а ведро опускает на землю. Механическую работу совершает мускульная сила фермера, поднявшего ведро массой, например, Работа, мощность и энергия в физике - виды, формулы и определения с примерами = 2,0 кг и картофель массой m = 10,0 кг на высоту h = 1,5 м. Какая работа здесь является полезной?

Работа, мощность и энергия в физике - виды, формулы и определения с примерами

Цель фермера — погрузить в кузов картофель. Исходя из этого, полезной работой является работа по подъему картофеля: Работа, мощность и энергия в физике - виды, формулы и определения с примерами А вот работа но подъему самого ведра не является полезной: Работа, мощность и энергия в физике - виды, формулы и определения с примерами Вся же совершенная (полная работа) равна:

Работа, мощность и энергия в физике - виды, формулы и определения с примерами

Какую долю составляет полезная работа от совершенной?

Работа, мощность и энергия в физике - виды, формулы и определения с примерами

Обозначим отношение Работа, мощность и энергия в физике - виды, формулы и определения с примерами буквой Работа, мощность и энергия в физике - виды, формулы и определения с примерами (эта) и назовем коэффициентом полезного действия (КПД). Тогда Работа, мощность и энергия в физике - виды, формулы и определения с примерами

КПД, как правило, выражают в процентах. 

Работа, мощность и энергия в физике - виды, формулы и определения с примерами

Таким образом, КПД (эффективность работы) в данном случае равен 83 %.

Рассмотрим еще один пример. Дети разгоняют санки, действуя силой F в направлении их движения (рис. 222). Совершенная (полная) работа здесь Работа, мощность и энергия в физике - виды, формулы и определения с примерами Цель детей — увеличить скорость движения санок. Но на санки действует еще сила трения скольжения Работа, мощность и энергия в физике - виды, формулы и определения с примерами Она тормозит движение санок. Значит, работа детей по преодолению силы трения не является полезной:

Работа, мощность и энергия в физике - виды, формулы и определения с примерами

Работа, мощность и энергия в физике - виды, формулы и определения с примерами

Полезной же работой была

Работа, мощность и энергия в физике - виды, формулы и определения с примерами

Тогда доля полезной работы (КПД)

Работа, мощность и энергия в физике - виды, формулы и определения с примерами

Физическая величина, равная отношению полезной работы к совершенной (полной), называется коэффициентом полезного действия.

А могут ли механизм, машина, человек работать так, чтобы КПД = 100 %, т. е. чтобы вся совершенная работа была полезной?

Ученые неоднократно пытались создать такую машину (рис. 223), но все попытки оказались безуспешными. (Самостоятельно познакомьтесь в Интернете или справочной литературе с информацией о вечном двигателе.) В работе любой машины, механизма всегда есть неполезная работа, идущая на преодоление трения, сопротивления. А значит, КПД всегда меньше 100 %. А вот сделать неполезную работу минимальной означает повысить КПД.

Работа, мощность и энергия в физике - виды, формулы и определения с примерами

Главные выводы:

  1. Совершенная (т. е. полная) механическая работа всегда больше полезной.
  2. КПД показывает, какую долю составляет полезная работа от всей совершенной.
  3. Чем больше полезная работа, тем выше КПД.
  4. КПД всегда меньше 100 %.

Пример решения задачи:

При подъеме картофеля из хранилища глубиной h = 3,6 м подъемным устройством с КПД Работа, мощность и энергия в физике - виды, формулы и определения с примерами = 90 % совершена работа Работа, мощность и энергия в физике - виды, формулы и определения с примерами = 40 кДж. Сколько мешков картофеля массой Работа, мощность и энергия в физике - виды, формулы и определения с примерами = 40 кг каждый было поднято из хранилища? Примите Работа, мощность и энергия в физике - виды, формулы и определения с примерами

Дано:

Работа, мощность и энергия в физике - виды, формулы и определения с примерами

Работа, мощность и энергия в физике - виды, формулы и определения с примерами

Работа, мощность и энергия в физике - виды, формулы и определения с примерами

Работа, мощность и энергия в физике - виды, формулы и определения с примерами

Работа, мощность и энергия в физике - виды, формулы и определения с примерами

Работа, мощность и энергия в физике - виды, формулы и определения с примерами

Решение:

Зная совершенную работу и КПД, можно найти полезную работу по подъему мешков картофеля:

Работа, мощность и энергия в физике - виды, формулы и определения с примерами

Полезная работа — это работа подъемного устройства по преодолению силы тяжести, действующей на картофель:

Работа, мощность и энергия в физике - виды, формулы и определения с примерами

Масса Работа, мощность и энергия в физике - виды, формулы и определения с примерами где N — число мешков картофеля. Тогда Работа, мощность и энергия в физике - виды, формулы и определения с примерами откуда

Работа, мощность и энергия в физике - виды, формулы и определения с примерами

Ответ: N = 25 мешков.

Мощность и единицы мощности

Приобретая автомобиль (рис. 226), газонокосилку, микроволновую печь (рис. 227) и др., человек интересуется их мощностью. Именно мощность является паспортной характеристикой машин и механизмов. Что же такое мощность? Почему так важно ее знать?

Работа, мощность и энергия в физике - виды, формулы и определения с примерами

Работа, мощность и энергия в физике - виды, формулы и определения с примерами

Рассмотрим пример. Человек лопатой копает яму для погреба в течение нескольких дней. Такую же яму экскаватор (рис. 228) выкопает за несколько минут. Работа выполняется одинаковая. Одинаковая масса грунта поднимается на одну и ту же высоту. Но быстрота совершения работы человеком и экскаватором разная. За единицу времени экскаватор выполняет во много раз большую работу, чем человек. Для описания быстроты совершения работы вводится мощность.

Работа, мощность и энергия в физике - виды, формулы и определения с примерами

Физическая величина, равная отношению работы к промежутку времени, за который эта работа совершена, называется мощностью. Обозначается мощность буквой Р.

Работа, мощность и энергия в физике - виды, формулы и определения с примерами

За единицу мощности в СИ принимается мощность, при которой действующая на тело сила за время t = 1 с совершает работу А = 1 Дж. Эта единица мощности называется ватт (Вт) в честь английского изобретателя Дж. Уатта. Для измерения больших мощностей используют кратные единицы: гектоватт (гВт), киловатт (кВт), мегаватт (МВт). Обратите внимание:

Работа, мощность и энергия в физике - виды, формулы и определения с примерами

Для малых мощностей употребляются дольные единицы — милливатт (мВт), микроватт (мкВт): 

Работа, мощность и энергия в физике - виды, формулы и определения с примерами

В быту часто необдуманно единицу мощности киловатт принимают за единицу работы. Но работа Работа, мощность и энергия в физике - виды, формулы и определения с примерами из чего следует, что единицей работы может быть только киловатт-час (кВт • ч), но не киловатт (кВт). Выразим мощность через другие единицы — силу и скорость. Мощность Работа, мощность и энергия в физике - виды, формулы и определения с примерами но работа Работа, мощность и энергия в физике - виды, формулы и определения с примерами путь Работа, мощность и энергия в физике - виды, формулы и определения с примерами Тогда

Работа, мощность и энергия в физике - виды, формулы и определения с примерами

Мощность пропорциональна силе, совершающей работу, и скорости движения. Тогда при постоянной мощности чем меньше скорость, тем больше сила. Вот почему водитель, трогаясь с места или поднимаясь в гору (рис. 229), когда требуется большая сила, едет на малой скорости. Тем самым он увеличивает силу тяги двигателя автомобиля.

Работа, мощность и энергия в физике - виды, формулы и определения с примерами

Главные выводы

  1. Мощность — физическая величина, характеризующая быстроту совершения работы.
  2. Единицей мощности в СИ является 1 ватт.
  3. Одинаковую мощность можно получить либо при большой скорости и небольшой силе, либо при малой скорости и большой силе.

Для любознательных

В автомобилестроении по традиции используют старинную единицу мощности — лошадиную силу (л. с.). С помощью рисунка сформулируйте самостоятельно определение мощности в 1 лошадиную силу.

Работа, мощность и энергия в физике - виды, формулы и определения с примерами

Запишем связь 1 л. с. и ватта: 1 л. с. = 736 Вт.

В этих внесистемных единицах мощность первого белорусского трактора МТЗ-2 (1953 г.) была равна 37 л. с. Освоенный в 2010 г. трактор «Беларус-3023» имеет двигатель мощностью 300 л. с. Переведите эти знамения мощности в единицы СИ самостоятельно и сравните их.

Работа, мощность и энергия в физике - виды, формулы и определения с примерами

Пример решения задачи:

На уроке физкультуры мальчик массой m = 40 кг поднялся по канату на высоту h = 5,0 м за промежуток времени t = 10 с. Определите среднюю мощность, развиваемую мальчиком при подъеме. Коэффициент Работа, мощность и энергия в физике - виды, формулы и определения с примерами

Дано:

Работа, мощность и энергия в физике - виды, формулы и определения с примерами

Работа, мощность и энергия в физике - виды, формулы и определения с примерами

Решение:

При подъеме по канату работа мускульной силы рук идет на преодоление силы тяжести.

Работа, мощность и энергия в физике - виды, формулы и определения с примерами

Тогда Работа, мощность и энергия в физике - виды, формулы и определения с примерами

Работа, мощность и энергия в физике - виды, формулы и определения с примерами

Ответ: P = 0, 20 кВт.

Кинетическая энергия

Энергия — одно из наиболее важных и сложных понятий. Причем не только в физике, но и в других науках. А что же такое кинетическая энергия?

Рассмотрим два примера. Шайба, попадая в сетку ворот (рис. 230), прогибает ее. Молот для забивания свай (рис. 231), падая на сваю, загоняет ее в землю на некоторую глубину. Чтобы сильнее прогнуть сетку или глубже забить сваю, шайба и молот должны иметь большую скорость. И шайба, и молот совершили работу. При этом скорость их движения изменилась (уменьшилась до нуля). Совершенные ими работы были разными, даже если предположить, что скорости движения были одинаковыми. Но массы молота и шайбы не равны.

Работа, мощность и энергия в физике - виды, формулы и определения с примерами

Работа, мощность и энергия в физике - виды, формулы и определения с примерами

Если тело способно совершить работу, то оно обладает энергией. В физике энергию движущегося тела называют кинетической (от греч. kinetikos — приводящий в движение). Кинетическая энергия обозначается буквой К (или Работа, мощность и энергия в физике - виды, формулы и определения с примерами) и измеряется в СИ в тех же единицах, что и работа, т. е. в джоулях.

Большая кинетическая энергия движущихся тел — камня, автомобиля, железнодорожного состава (рис. 232), метеорита и др. — означает, во-первых, что при разгоне их до данной скорости разгоняющей силой была совершена большая работа и, во-вторых, при их остановке тормозящей силой будет совершена такая же большая работа.

Работа, мощность и энергия в физике - виды, формулы и определения с примерами

Из примеров следует, что кинетическая энергия зависит от массы тела и скорости его движения. Какой является эта зависимость?

Опыты показывают, что кинетическая энергия прямо пропорциональна массе тела и квадрату скорости его движения:

Работа, мощность и энергия в физике - виды, формулы и определения с примерами

Увеличение скорости движения тела, например в 4 раза, приводит к возрастанию кинети- Обратите внимание! ческой энергии в 16 раз. Об этом должны всегда помнить водители и пешеходы.

Главные выводы:

  1. Кинетическая энергия выражает способность движущихся тел совершать работу.
  2. Кинетическая энергия, как и работа, измеряется в джоулях.
  3. Кинетическая энергия тела зависит от его массы и скорости.
  4. Изменить (увеличить или уменьшить) кинетическую энергию тела можно только путем совершения работы (положительной или отрицательной).

Пример решения задачи:

Скорость движения груженого автомобиля массой m = 4,0 т увеличилась от Работа, мощность и энергия в физике - виды, формулы и определения с примерами до Работа, мощность и энергия в физике - виды, формулы и определения с примерами на пути s = 25 м. Определите силу тяги двигателя автомобиля и работу, которую совершила эта сила. Сопротивление движению не учитывать.

Дано:

Работа, мощность и энергия в физике - виды, формулы и определения с примерами

Работа, мощность и энергия в физике - виды, формулы и определения с примерами

Решение:

Чтобы увеличить кинетическую энергию от Работа, мощность и энергия в физике - виды, формулы и определения с примерами до Работа, мощность и энергия в физике - виды, формулы и определения с примерами сила тяги должна была совершить работу:

Работа, мощность и энергия в физике - виды, формулы и определения с примерами

Работа, мощность и энергия в физике - виды, формулы и определения с примерами

Но работа Работа, мощность и энергия в физике - виды, формулы и определения с примерами Отсюда Работа, мощность и энергия в физике - виды, формулы и определения с примерами

Ответ: Работа, мощность и энергия в физике - виды, формулы и определения с примерами

Потенциальная энергия

При разгоне любого тела (санок, автомобиля и др.) у него возникает способность совершить механическую работу — у движущегося тела появляется кинетическая энергия. А если тело неподвижно? Обладает ли оно способностью совершить работу?

Проведем два опыта. В первом поднимем и укрепим на нити над ящиком с песком гирю (рис. 235, а). Во втором между упором и шариком поместим предварительно сжатую и связанную ниткой пружину (рис. 235, б). Оба тела (гиря и пружина) неподвижны Работа, мощность и энергия в физике - виды, формулы и определения с примерами и не обладают кинетической энергией. Но и у гири, и у пружины есть возможность совершить работу. Для этого достаточно в обоих случаях пережечь нить. В физике говорят, что тела (поднятая гиря, взаимодействующая с Землей, и сжатая пружина) обладают потенциальной энергией (от лат. potentia — скрытая способность). Потенциальную энергию в СИ измеряют в тех же единицах, что и работу, — в джоулях.

Работа, мощность и энергия в физике - виды, формулы и определения с примерами

Важно понимать, что потенциальная энергия не появляется сама по себе. В этих опытах гиря была поднята над столом, пружина была сжата какой-то силой. Значит, чтобы тело запасло потенциальную энергию, необходимо совершить работу. Чем сильнее будет сжата пружина, чем выше будет поднято тело, тем больше у них будет запас потенциальной энергии. Тела, представленные на рисунке 236, уже обладают потенциальной энергией. У трамплина она вызвана прогибом (деформацией) доски, у мышеловки — закручиванием пружины, у лука — изменением расположения древка и тетивы. Из этих и других примеров следует, что потенциальная энергия — это энергия, обусловленная взаимным расположением взаимодействующих тел или частей тела (гири и Земли, стрелы и тетивы, звеньев пружины). Обозначается потенциальная энергия буквой П (или Работа, мощность и энергия в физике - виды, формулы и определения с примерами).

Работа, мощность и энергия в физике - виды, формулы и определения с примерами

Именно благодаря потенциальной энергии сжатой (закрученной) пружины работают механические часы, реле времени микроволновых печей, стиральных машин, движутся некоторые детские игрушки. Потенциальная энергия поднятой с помощью плотины воды заставляет работать гидроэлектростанции (рис. 237).

Работа, мощность и энергия в физике - виды, формулы и определения с примерами

Главные выводы:

  1. Неподвижные взаимодействующие тела (система тел) могут обладать способностью совершать механическую работу, а значит, потенциальной энергией.
  2. Значение потенциальной энергии зависит от взаимного расположения взаимодействующих тел (частей тела).
  3. Потенциальная энергия изменяется только при совершении работы.

Расчет потенциальной энергии

Кинетическая энергия тела, зависящая от его массы и скорости, выражается формулой Работа, мощность и энергия в физике - виды, формулы и определения с примерами Данная формула справедлива и для планеты Земля, мчащейся со скоростью Работа, мощность и энергия в физике - виды, формулы и определения с примерами по орбите вокруг Солнца, и для невидимого нашему глазу атома. Существует ли единая формула для расчета потенциальной энергии?

Рассмотрим отдельно два случая: потенциальную энергию притяжения поднятого над поверхностью Земли тела и потенциальную энергию деформированного тела.

В первом случае формулу для расчета потенциальной энергии легко вывести. Если тело массой m поднято относительно поверхности Земли на высоту h (рис. 238), то при его падении сила тяжести Работа, мощность и энергия в физике - виды, формулы и определения с примерами может совершить работу:

Работа, мощность и энергия в физике - виды, формулы и определения с примерами

Это и есть потенциальная энергия поднятого тела:

Работа, мощность и энергия в физике - виды, формулы и определения с примерами

Работа, мощность и энергия в физике - виды, формулы и определения с примерами

Значение потенциальной энергии относительно. Так, относительно пола потенциальная энергия светильника (рис. 239) массой m = 1,0 кг, центр тяжести которого расположен на высоте Работа, мощность и энергия в физике - виды, формулы и определения с примерами от пола, равна:

Работа, мощность и энергия в физике - виды, формулы и определения с примерами

Относительно потолка Работа, мощность и энергия в физике - виды, формулы и определения с примерами она равна:

Работа, мощность и энергия в физике - виды, формулы и определения с примерами

Работа, мощность и энергия в физике - виды, формулы и определения с примерами

Поэтому, приводя значение потенциальной энергии, необходимо указывать уровень, относительно которого она задана, — нулевой уровень потенциальной энергии (это может быть, к примеру, поверхность пола, потолка, стола и т. д.).

Гораздо сложнее дело обстоит с расчетом потенциальной энергии деформированного тела. Мы можем растянуть или сжать пружину, изогнуть или закрутить ее (рис. 240). Потенциальная энергия у пружины будет в каждом из этих случаев. И чем больше упругая деформация, тем больше потенциальная энергия пружины. В данном примере расчет потенциальной энергии придется вести по различным формулам. Более детально с этим вы будете знакомиться в 9-м классе.

Работа, мощность и энергия в физике - виды, формулы и определения с примерами

Главные выводы:

  1. Потенциальная энергия притяжения тела к Земле зависит от массы тела и высоты его подъема над нулевым уровнем энергии.
  2. Значение потенциальной энергии тела зависит от выбора нулевого уровня энергии.
  3. Потенциальная энергия деформированного тела зависит от величины деформации.

Пример решения задачи:

Парафиновый однородный кубик с длиной ребра а = 10 см лежит на столе на высоте Работа, мощность и энергия в физике - виды, формулы и определения с примерами = 0,80 м от пола. Определите потенциальную энергию кубика относительно поверхностей: а) пола; б) стола. Какую работу нужно совершить, чтобы поднять кубик с пола на стол? Коэффициент Работа, мощность и энергия в физике - виды, формулы и определения с примерами

Дано:

Работа, мощность и энергия в физике - виды, формулы и определения с примерами

Работа, мощность и энергия в физике - виды, формулы и определения с примерами

Решение:

Потенциальная энергия кубика относительно поверхности пола (рис. 241) определяется положением его центра (точки O):

Работа, мощность и энергия в физике - виды, формулы и определения с примерами

Масса кубика Работа, мощность и энергия в физике - виды, формулы и определения с примерами объем Работа, мощность и энергия в физике - виды, формулы и определения с примерами тогда:

Работа, мощность и энергия в физике - виды, формулы и определения с примерами

Работа, мощность и энергия в физике - виды, формулы и определения с примерами

Работа, мощность и энергия в физике - виды, формулы и определения с примерами

Потенциальная энергия кубика относительно поверхности стола:

Работа, мощность и энергия в физике - виды, формулы и определения с примерами

Работа, мощность и энергия в физике - виды, формулы и определения с примерами

Работа по подъему кубика на высоту Работа, мощность и энергия в физике - виды, формулы и определения с примерами равна изменению его потенциальной энергии. Получаем:

Работа, мощность и энергия в физике - виды, формулы и определения с примерами

Ответ: Работа, мощность и энергия в физике - виды, формулы и определения с примерами

Закон сохранения механической энергии

Кинетическая и потенциальная энергии — это два вида механической энергии. Связаны ли они друг с другом? И если да, то в чем выражается эта связь?

Проследим за движением брошенного вверх металлического шарика (рис. 243). В нижней точке траектории сила действия руки на шарик сообщает ему кинетическую энергию. Шарик движется вверх. Скорость его движения, а значит, и кинетическая энергия уменьшаются. Но исчезает ли кинетическая энергия бесследно? Поднимаясь выше, шарик приобретает все большую потенциальную энергию (вспомните: Работа, мощность и энергия в физике - виды, формулы и определения с примерами). В верхней точке скорость и кинетическая энергия шарика равны нулю, а потенциальная максимальна. Значит, в рассмотренном примере происходит превращение энергии из одного вида (кинетической) в другой (потенциальную). При возвращении шарика обратно снова будет идти превращение энергии: с уменьшением высоты (и потенциальной энергии) увеличивается скорость движения шарика (и кинетическая энергия).

Работа, мощность и энергия в физике - виды, формулы и определения с примерами

Если сопротивление воздуха мало (и им можно пренебречь), брошенный вверх шарик возвращается назад практически с такой же, как в момент бросания, скоростью и кинетической энергией.

А каким будет значение механической энергии шарика в промежуточных точках? Например, на высоте Работа, мощность и энергия в физике - виды, формулы и определения с примерами (рис. 243)? При подъеме шарика на высоту Работа, мощность и энергия в физике - виды, формулы и определения с примерами его кинетическая энергия уменьшилась, но при этом появилась потенциальная энергия. А чему равна их сумма, т. е. полная механическая энергия? Данный и подобные опыты и расчеты показывают, что если сил сопротивления нет, то полная механическая энергия тела (системы тел), равная сумме кинетической и потенциальной энергий Работа, мощность и энергия в физике - виды, формулы и определения с примерами сохраняется. Данное утверждение о постоянстве механической энергии в физике называют законом сохранения механической энергии.

Если силами трения или сопротивления движению нельзя пренебречь, этот закон не выполняется. Заменим в опыте металлический шарик на пенопластовый брусок такой же массы (рис. 244). Мы увидим, что даже при большей, чем у металлического шарика, начальной скорости он не поднимется на такую же высоту и вернется назад с заметно меньшей скоростью. Убывает кинетическая энергия движущейся по горизонтальной поверхности льда шайбы, но потенциальная энергия взамен не появляется. За счет кинетической энергии шайбы совершается работа против сил трения.

Работа, мощность и энергия в физике - виды, формулы и определения с примерами

В заключение заметим, что явление превращения энергии из одного вида в другой человек научился использовать в практических целях. Энергия падающей воды приводит в действие водяные мельницы и гидроэлектростанции. В Республике Беларусь успешно реализуется государственная программа использования энергии рек. Важная роль в ней отводится таким рекам, как Неман и Западная Двина. Па Немане работает Гродненская ГЭС мощностью 17 МВт. Установленная мощность Витебской ГЭС на Западной Двине — 40 МВт.

Кинетическую энергию ветра человек с давних времен начал использовать с помощью паруса (рис. 245), затем стал применять в ветряных мельницах. В последние годы в нашей стране начато сооружение ветроэлектростанций (рис. 246). Они уникальны тем, что не оказывают вредного воздействия на окружающую среду. Во многих странах успешно используют энергию приливов и отливов вод морей и океанов. Там созданы приливные электростанции.

Работа, мощность и энергия в физике - виды, формулы и определения с примерами

Работа, мощность и энергия в физике - виды, формулы и определения с примерами

Главные выводы:

  1. Кинетическая и потенциальная энергии взаимо-превращаемы.
  2. При отсутствии сил трения и сопротивления движению полная механическая энергия тела (системы тел) сохраняется.
  3. Закон сохранения механической энергии не выполняется, если силами трения (сопротивления) нельзя пренебречь.
  • Заказать решение задач по физике

Пример решения задачи:

Камень бросили вертикально вверх со скоростью Работа, мощность и энергия в физике - виды, формулы и определения с примерами На какой высоте от точки бросания кинетическая энергия камня будет в 4 раза меньше его потенциальной энергии? Сопротивлением движению камня пренебречь. Коэффициент Работа, мощность и энергия в физике - виды, формулы и определения с примерами

Дано:

Работа, мощность и энергия в физике - виды, формулы и определения с примерами

Работа, мощность и энергия в физике - виды, формулы и определения с примерами

Решение:

За нулевой уровень потенциальной энергии примем уровень O — O, проходящий через точку бросания камня (рис. 247). Значит, Работа, мощность и энергия в физике - виды, формулы и определения с примерами

Полная механическая энергия камня в точке бросания 1:

Работа, мощность и энергия в физике - виды, формулы и определения с примерами 

Работа, мощность и энергия в физике - виды, формулы и определения с примерами

Полная механическая энергия камня в точке 2:

Работа, мощность и энергия в физике - виды, формулы и определения с примерами

По условию Работа, мощность и энергия в физике - виды, формулы и определения с примерами Значит,

Работа, мощность и энергия в физике - виды, формулы и определения с примерами

Работа, мощность и энергия в физике - виды, формулы и определения с примерами

Ответ: Работа, мощность и энергия в физике - виды, формулы и определения с примерами

Энергия и работа

Энергия – эта количественная мера различных форм движения и взаимодействия (по гречески слово «энергия» означает действие). Энергия в зависимости от вида движения в природе проявляется по-разному. Например, механическая, тепловая, электромагнитная, ядерная энергия и другие. В результате взаимодействия энергия одного вида превращается в энергию другого вида. Однако во всех этих процессах энергия, переданная от одного тела второму (независимо от ее вида), будет равна энергии, полученной вторым телом от первого.

Как известно из второго закона Ньютона, чтобы изменить механическое движение тела на него должны подействовать другие тела. Иначе говоря, среди этих тел происходит обмен энергиями. Для описания такого обмена энергии в механике введено понятие механическая работа, которую принято обозначать буквой Работа, мощность и энергия в физике - виды, формулы и определения с примерами.

Механическая работа. Величина, равная скалярному произведению силы на перемещение в направлении действия силы, называется механической работой, т.е. 

Работа, мощность и энергия в физике - виды, формулы и определения с примерами

Здесь: Работа, мощность и энергия в физике - виды, формулы и определения с примерами – угол между силой Работа, мощность и энергия в физике - виды, формулы и определения с примерами и перемещением Работа, мощность и энергия в физике - виды, формулы и определения с примерамиs (рисунок 3.1). 
Если учитывать, что Работа, мощность и энергия в физике - виды, формулы и определения с примерами, то уравнение (3.1) примет вид:

Работа, мощность и энергия в физике - виды, формулы и определения с примерами

Здесь Работа, мощность и энергия в физике - виды, формулы и определения с примерами – проекция силы в направлении смещения.
Основываясь на выражении (3.2), можно сделать следующий вывод:
если Работа, мощность и энергия в физике - виды, формулы и определения с примерами, то Работа, мощность и энергия в физике - виды, формулы и определения с примерами – работа силы положительна, направление силы и смещение совпадают;
если Работа, мощность и энергия в физике - виды, формулы и определения с примерами, то Работа, мощность и энергия в физике - виды, формулы и определения с примерами – работа силы отрицательная, направления силы и смещения противоположны;
если Работа, мощность и энергия в физике - виды, формулы и определения с примерами, то Работа, мощность и энергия в физике - виды, формулы и определения с примерами – работа, выполненная силой, равна нулю, направление силы будет перпендикулярным к направлению смещения.
Работа считается аддитивной (аддитив – по-латински означает суммарный) величиной (в физике аддитивность величины означает, что величина, относящаяся к системе в целом, равна сумме величины, относящихся к ее составным частям).
Если на тело действует несколько сил, то будет:

Работа, мощность и энергия в физике - виды, формулы и определения с примерами

тогда полная работа равна работе, выполненной равнодействующей сил.

Работа, мощность и энергия в физике - виды, формулы и определения с примерами

или

Работа, мощность и энергия в физике - виды, формулы и определения с примерами

Единица работы. Единица измерения работы в системе СИ – Джоуль (Дж):

Работа, мощность и энергия в физике - виды, формулы и определения с примерами

В качестве единицы работы в СИ принята работа выполненная силой 1Н при смещении тела на 1 м.
Работа силы тяжести. На поверхности Земли на тело действует сила тяжести со стороны Земли, равная Работа, мощность и энергия в физике - виды, формулы и определения с примерами. При перемещении тела из точки Работа, мощность и энергия в физике - виды, формулы и определения с примерами на высоте Работа, мощность и энергия в физике - виды, формулы и определения с примерами от поверхности Земли в точку Работа, мощность и энергия в физике - виды, формулы и определения с примерами на высоте Работа, мощность и энергия в физике - виды, формулы и определения с примерами от поверхности Земли, смещение тела равно: Работа, мощность и энергия в физике - виды, формулы и определения с примерами (рис. 3.2).

Работа, мощность и энергия в физике - виды, формулы и определения с примерами

Здесь выполненная силой тяжести работа выражается следующей формулой:

Работа, мощность и энергия в физике - виды, формулы и определения с примерами

Здесь: Работа, мощность и энергия в физике - виды, формулы и определения с примерами – вес тела, Работа, мощность и энергия в физике - виды, формулы и определения с примерами – его масса, Работа, мощность и энергия в физике - виды, формулы и определения с примерами – ускорение свободного падения, Работа, мощность и энергия в физике - виды, формулы и определения с примерами – расстояние между уровнями Работа, мощность и энергия в физике - виды, формулы и определения с примерами и Работа, мощность и энергия в физике - виды, формулы и определения с примерами по вертикали.
Работа, выполненная силой тяжести, не зависит от формы пути, зависит только от высоты спуска. Поэтому работа, выполненная под действием силы тяжести, зависит не от формы  траектории, а от начального и конечного состояний. Такая сила называются потенциальной или консервативной. Поле такой силы называется потенциальным полем.

При движении тела вниз из-за соответствия направления силы тяжести и смещения выполненная работа будет положительной, при движении вверх из-за противоположности направлений работа будет отрицательной. Поэтому в случае, когда тело под воздействием силы тяжести смещено и вернулось обратно, выполненная общая работа равняется нулю.

Полной механической энергией системы называется сумма кинетической и потенциальной энергии системы. Например, полная механическая энергия тела массой Работа, мощность и энергия в физике - виды, формулы и определения с примерами, двигающегося со скоростью Работа, мощность и энергия в физике - виды, формулы и определения с примерами относительно Земли на высоте Работа, мощность и энергия в физике - виды, формулы и определения с примерами от поверхности Земли:

Работа, мощность и энергия в физике - виды, формулы и определения с примерами

Полная механическая энергия системы остается неизменной с течением времени:
 

Работа, мощность и энергия в физике - виды, формулы и определения с примерами

Возможны лишь превращения потенциальной энергии и кинетическую и обратно. Выражение (3.5) представляет собой закон сохранения механической энергии.

Проведенные многочисленные эксперименты, теоретические выводы подтвердили строгое соблюдение закона сохранения энергии. 
В природе постоянно происходят превращения одного вида энергии в другой (например, механическая энергия переходит в тепловую энергию). Поэтому этот закон также называют законом сохранения и превращения энергии. Этот закон является основным законом природы и действителен не только для макроскопических, но и микроскопических систем.

Энергия никогда не исчезает, ниоткуда не появляется, она может только преобразовываться из одного вида в другой. 

В закрытых системах полная энергия сохраняется.

Например, потенциальная энергия тела, падающего с высоты Работа, мощность и энергия в физике - виды, формулы и определения с примерами, зависит от его веса и абсолютно не зависит от времени проведения экспериментов.
Коэффициент полезного действия. Введена величина, показывающая, какая часть израсходованной энергии машин и двигателей превращается в полезную работу.

Отношение полезной работы к полной работе называется коэффициентом полезного действия (КПД) и обозначается буквой Работа, мощность и энергия в физике - виды, формулы и определения с примерами
Если полезную работу обозначить Работа, мощность и энергия в физике - виды, формулы и определения с примерами, полную работу Работа, мощность и энергия в физике - виды, формулы и определения с примерами, тогда формулу КПД можно записать в виде:

Работа, мощность и энергия в физике - виды, формулы и определения с примерами

КПД не может быть больше единицы (100%). В машинах и двигателях в результате работы силы трения часть полной энергии расходуется и поэтому КПД всегда меньше единицы.

Рассмотрим наклонную плоскость и выполненную работу при подъеме тела вверх. По «золотому правилу» механики, во сколько раз выигрываем в силе, во столько раз проиграем в расстоянии. Но из-за увеличения расстояния смещения не меняется выполненная работа.

Работа, мощность и энергия в физике - виды, формулы и определения с примерами

Рассмотрим груз с весом Работа, мощность и энергия в физике - виды, формулы и определения с примерами на наклонной плоскости длиной Работа, мощность и энергия в физике - виды, формулы и определения с примерами, высотой Работа, мощность и энергия в физике - виды, формулы и определения с примерами (рис. 3.3). Здесь на тело действует сила трения Работа, мощность и энергия в физике - виды, формулы и определения с примерами, параллельная наклонной поверхности тянущая вверх сила Работа, мощность и энергия в физике - виды, формулы и определения с примерами, перпендикулярно направленная к наклонной плоскости Работа, мощность и энергия в физике - виды, формулы и определения с примерами и противоположно направленная перпендикулярно к поверхности сила Работа, мощность и энергия в физике - виды, формулы и определения с примерами (реактивная сила поверхности).
Если не учитывать силу трения, получим уравнение:

Работа, мощность и энергия в физике - виды, формулы и определения с примерами

Однако с учетом силы трения,

Работа, мощность и энергия в физике - виды, формулы и определения с примерами

Тогда Работа, мощность и энергия в физике - виды, формулы и определения с примерами пишется в следующем виде:

Работа, мощность и энергия в физике - виды, формулы и определения с примерами

Коэффициент полезного действия:

Работа, мощность и энергия в физике - виды, формулы и определения с примерами

Сила притяжения, действующая на груз, равна:
 

Работа, мощность и энергия в физике - виды, формулы и определения с примерами

  • Движение и силы
  • Давление в физике
  • Строение вещества в физике
  • Физическое тело и вещество в физике
  • Золотое правило механики
  • Потенциальная энергия
  • Кинетическая энергия
  • Закон сохранения и превращения механической энергии

Понравилась статья? Поделить с друзьями:
  • Как найти подлежащее выражено сочетанием слов
  • Не работает windows на компьютере как исправить ошибку
  • Как найти циклическую частоту гармонических колебаний точки
  • Перекос рта как исправить
  • Как найти клиентов для ремонта окон